Вы находитесь на странице: 1из 39

Official

LSAT PrepTest

Form 4LSN111

December 2014
74
© 2014 by Law School Admission Council, Inc.

All rights reserved. No part of this work, including information, data, or other portions of the work published in
electronic form, may be reproduced or transmitted in any form or by any means, electronic or mechanical, including
photocopying, recording, or by any information storage and retrieval system, without permission of the publisher. For
information, write: Communications, Law School Admission Council, 662 Penn Street, Box 40, Newtown, PA, 18940-
0040.

LSAT is a registered trademark of the Law School Admission Council, Inc.

All actual LSAT questions printed within this work are used with the permission of Law School Admission Council,
Inc., Box 2000, Newtown, PA 18940, the copyright owner. LSAC does not review or endorse specific test preparation
materials or services, and inclusion of licensed LSAT questions within this work does not imply the review or
endorsement of LSAC.

2
THE PREPTEST

• The PrepTest........................................................................................................................................................................3

• Section I.........................................................................................................................................................................4

• Section II......................................................................................................................................................................12

• Section III.....................................................................................................................................................................20

• Section IV.....................................................................................................................................................................28

• Acknowledgments.............................................................................................................................................................36

• Computing Your Score......................................................................................................................................................37

• Answer Key.........................................................................................................................................................................38

3
74

1 -4-
1 1
SECTION I
Time—35 minutes
25 Questions

Directions: The questions in this section are based on the reasoning contained in brief statements or passages. For some
questions, more than one of the choices could conceivably answer the question. However, you are to choose the best answer; that
is, the response that most accurately and completely answers the question. You should not make assumptions that are by
commonsense standards implausible, superfluous, or incompatible with the passage. After you have chosen the best answer,
blacken the corresponding space on your answer sheet.

�� �������� ������ �� ����������� ���� ������� �����’� �� ���������� ��� ���������� ������ ��� ���� ��� ���� ��
������ ������� ���� �� ���� ������ ������ ����� �������� ��� ������ �������� ���� ������� �������� �����
�������� ���������� ������� ����������� ��� ������ ��� ������ �������� ��� ��������� �� ��� ����������
������ ��������� ��� ����� ���’� ����� ��� ������ ����� ��� ������� ������� ����� ������� �� ���� ���� ��
�� ����� ������ ��� ������� ���� ������� �������� ���� ��� ������� ������� ��� ���������� ���������� ��
�����’� ����� �� ���� �������� ���� �� � ������ ����������
����� ��� �� ��� ��������� ���� ��������� ��������� ��� ��� �������� �� ��� ��������� ���� ������� �������� ��
�������� ������ ����� ��� �� ��� ��������� �����������
��� �� ���� ���� ���� �� ������� � ����� ��� ����� ��� ��� ����������� ������ ����� ���� �� ����� �������
������ �� ��������� �������� ���� �� ������������
��� ��� ���� �������� ��� ������ ���������� �� ����� ���������
���������� ��� ���� ������� ���������� ��� ��������� ���������� �������� ���� �� ������
��� ����� ����� �� ���������� ���� ��� ����������� �� ���� �������� ������� �� ��������
������������ ��� ���� ����������� ���������� ��� �� ���������� ������� ������ �� ��������
��� ����� ������� ��� ��������� ��� �������� ��������� �� ������� ������� ��� ��� ��������
�������� ���� �� ������� ��� ����������� ������ �������� ��� ������ ��������
��� �������� ��� ������ �� ���������� ���� ����� �� �� ��������� �� ��� ����������
���� ���� ���� ���� �� ����� ��� ����� ���� ��� �� ����� ���� ������� ������� ���� �� ��� ����� ��
������� ���� �� ������ ���������� �� ��� �������

�� �������������� ��� ��� ��� ���������’� �����������


������ �� ����� ������� ���� ��� ������ �������
�� ��� ������ �� ��������� ��� ���� ���� ���
���������� �� ��������� �� ��� ��������� �� � �����
������� �� � ������ ������ ���� ��� ����� �� � GO ON TO THE NEXT PAGE.
����� ������ �� ������� ��� ������� �� � �������
���� �� � ������ ���������� �� � ����� �����
����� �� �������� �� ��������� ��� ������� ����� ��
����� ������ �� ��� ������� ��� ����������� ���
������ ��� ���������’� ������� ��� ���� ������
�� � ������� ���� ��� ��� �� �������
����� ��� �� ��� ���������� �� ����� ���� ��� ���� �������
��� ��� �������������’� �����������
��� ������ �������� �� �������� ������� ���� �����
�� ��� ��������� ���� ���� ���� ���������
�����������
��� ��� ������������ �� ��� ������ �� ������ ���
����� ���� � ������� ����� ��� ��������� ���
������������
��� ����� �������� ���� �������� ���� �� ����
����� �� ������������ �� ������� �������
��� ��� ��������� ������� �� ������ ��� ������
����� �� ���� �� ��� ������� �� ��� ����������
��� ��� ���������’� ����������� ��� ������������
���������� ������� ������� ������ ���� �������
���� �� ��� ����������
74

1 1 -5-
1
�� ���������� ������������ ���� �� ����� ���������� ������ �� ����� ���������� ����� �� � ������� ������ �� ��� ����
���������� ���������� ����� ��������� ������ � ���������� ���� � �������� �� ��� ����� ������� ��� �������
����������� ����������� �� ������� �� ����� ����� ��������� ��� ��������� ���� ��� ������ ���� ������
��� ������� ����� ��� ������� ���� ���� ��� ��������� ����� ��� ���� ������ ������ ������� ��� �����
��������’� ���� ���� ����� �� ���� ��������� � ���� �� ���������� ����� �� �� ������ ��������� �� ���� ���
������������ ����� ������������ ���� �� �������� ������ ��������� ����� ��� ���� ������� �������� �����
������� ����� ���� ������� ���� ����� ��� ������� �� �� ���� ����� ����� �� ����� ��������� ����� ����
�� �� ��� ��������’� ���� �������� ����� � ���� �� ���������� ������� �� ���������� ������
��������� �������������
����� ��� �� ��� ���������� �� ����� ���� ���������
����� ��� �� ��� ���������� �� ����� ���� ����� �� ������� ������� ��� ���������
��� ���������’� �����������
��� ��� ������ �� ��� �������� �� ���������� ����� ��
��� ������������ �� ��� ���� ��� ������� �� �������� ������ ������������� ���� ����� �� ������
��� ����������� �� ����� ������ ��� ���������� ����� �� ���� ��������� ���� ���� �� ���
��� ��������� ���� �� ����� ����� ��� ��� ���� ���� ��������� ���� ����� �� ����������
���� ������� �� ������� ��� ��������� �� ����� ��� ���� ���������� ������� ��� �������� ����
���������� ���������� ������ ��� ����� �� ������ �������� ���
��� ������������ ����� ���� ���� ����������� ���� ������� ��������� ���� ���������
���������� ���� � �������� ����� ���� �� ������ ��� ��� �������� �� ���������� ����� ����� ��
�� ���� ���� ���������� ���� ��� ����� ���� �� ���������� �� ������ �� ��� ���� ���� �������
���� ��� ������� ��������� ��� ������������
��� ��� ����������� �� ��� ��� ���� �������� �� ��� ��� ���� ������ ��� �������� �� ���������� ����� ���
���������� ������ �������� ���� ������ �� �� ����������� ������� ���� �������� �������� ���
������ �� ��������� �������� �� � ��������� ���� ������� ��������� �� ����� ���������� ������
����������
��� ������� ���� �� ��� ���������� ������ �������� �� � �������� ������ �� ������� ��� ���������������� ����
���� �� ����� ��� ������� �������� ������� �� � ����� ������ ����� ��� ����������� �� ��� ���������
����� ������ ��� �������� ��� ���������������� ����� ����
�� ������� ��������� �� ����������� ���������� ���������� ��������� ������� ��� ���� ��� �������� ���� ��� ���������
������ ���� �������� ���������� ��������� �� ��� ��� ��� ���� �������� ���� ��� ��� ������ ��� ������� ���
����������� �� ���� ����� ��������� �������� ����� �� �������� ��� ���� �������� �������� ����� ������ ����
���� �� ��� ������ ����� � ������ ���� �������� ��� ��� �������� ��� �������� ������ ��� ��� ������
����� ���� ���� �� ���������� ���� ����������� ���
��� �����������’� ������ �� ��� �������� ���� �������
�� ������ ������ ��������’� ��������� �������
�������� �� ����� ��� �� ��� ��������� �����������
������� �� �� �� ��������’� ������� �� ��������
������� ���� ���������� �� ��� ������� �� ����� ��� ���� ��� ������ �� ��� ��������� ������ �� �
��������� ���������� ���� ��������� ��� ���������� ������
�� ��� �������� ��� ������� ��� ����� �� ���
����� ��� �� ��� ��������� ���� ���������� ��������� �
���������
���� �� ��� ������’� ����������
��� ���� � �������� �� �������� ������� ���
��� �� ����� � ��������� ��������� ��� ���������’� ��������� ������ �� � ���������� ���� ����������
����� ������� �� �� � ���������� ��������� ��� ��� �������� ������ ������ ��� ���� ��������
����� �������� ������� ���� �� �� �� ���� ���� ����������
��� �� ��������� ���� ��������� �� ���� ������ �� ��� ��� � ����������� ������ ������ ��������� ����
������� ���� ����� �� �� �������� �� ��� ��������� � �������� ��� ��� ������� ���� ���������� ���
��� �� ������� �� �������� ������ �� ��� ������� ���� ���� ������
��� �������� ����� ����� ��� ��������� �� ��� ��� ���� �������� ��������� ����� � �������� ��
������ ������ ���� ��������� � ���� ���������� � ����������� ������ ������
��� �� �� ����� �� �� ������ �� ��� ����� �� ������� ��� �������� ������ ��� ������� ���� ������ ���
���� ������������ ��������� �� ��� ������� ������� ���������� ��� ��� ������������
������� ��� ���� � �������� �� �������� � ���� ����������
��� �� ����� ��� ���������� �� ������ ���� ��� ���� �������� ������ ������ ��� ����� ��������
������������ ���� ��� �������� ������� ���� ����� ��� �� ��� ������

GO ON TO THE NEXT PAGE.


74

1 -6-
1 1
�� ���������� ��� ��������� ����������� ���� �� ����� ������� ��� ���������� ������ ������ ��� �� ��������� ������� ���� ��
�� ��� ����� �� ��� ������’� �������� ��� �� ���� ���� ��� �� ��������� �� ��� ������������ ��� ����
�� ������ ��������� ���� ����� ������� ���� ������ ���� ��� ����� ��������� ��� ��� ��������� ���������
��������������� ��� ���� �������� ���� ����� ��� ��� ���� ��� ��� ����� �� �� �����������
��������� �� ��� ��� ������’� ������������ ��� ������������ ������� ������ ��� ��� ��� ���������
���������� ������ ���� ������ �� ������ ��������� ������� ���� ������� ��� ���� �� ��� ���������
�� ������ ����� �� ����� ��� ��������� �� ���
����� �� ��� ������� �� �� ������� �� � ��������� ����� ��� �� ��� ���������� �� ����� ���� ����� �� �������
������������ ��� ����� ����������� �� ��� ����������

����� ��� �� ��� ��������� �� �� ���������� ���� ��� ��� ����� ���� ����� �� ���� �� ��� ��������� ������
�������� ��������� ����� �� ����� ����������� ������� ��� ��������
����� � ��� ������� ������ ��� ��� ���������
��� ��������� �� ���������� ������’� �������� ����� ����������� ��������� �� ����������� �������������
���� ��� ������ �������� ����� ��� ��� ������’� ��� �������� ��� ������ �� ������� �� ���� ����
������������ ��� ���������� ��������� ��� ��� ���� �� ������� ������ �������������
��������� �� �������� ������������ ��� ������� �� ��� ���� ���� ��� ����
��� ��������� ����������� ���� ����� �� ��� ����� ���� ���������� ���� �� ����� �� ��� �������
�� ��� ������’� ������� ���� ������ ���� ��� ��� ������ �� ��� ���� ��� ������� �������������
���� ������ �� ��� ������’� ������������ ��� �� ��� ��� ��� ������� ��� ������� �� ������
���������� ��� ��� ��������� ��������� �� ��� ����� ��� ������ ��������� ���
��� ��� �������� �� ��� ������’� ������� ������������ �������� ���� �� ���� �� ������ ��������� �� �����
��� ���������� ������ ���� ���� ������ ���������� �� ��� �������
����������� ���� ������ ��������� �� ������ ��� ��� ������ ����� ��� ��� ���� �� ������������� ����
��� �� ���� �� ��� �� ��� ������ ��� ���� ��� ������ ���� ��� ������ ������� ������ �� �� ������ ���
�� ���������� ������� ��� ������� �� ��� ������’� ��� ���� �� �� � ����� ���� ������� ������������
������������ ��� ����������� ������
��� ������������� �� ��� ����� �� ��� ������’� ������� ��� ����� ������� ����� ��� ����� ��� �������� �� ���
������� ������� ��������������� ���� ����� �������� ����� ��� ���������� ����� ��� ���������� ����
���������� ��� ��� ���� ������� ��� ��� ����
�� ���� ������� ����������� ������� ����� � ����������� ������ �� �������� ������� �� �������������
��������� ��������� ���� �� ����� �������� ��� ���� ���� ��� ���������
��� ������ ����� �� ����� ����� ������� �� ���� ����
����� ��� ���� ������ �� ���� �� ��� ����� ��� � �����
������ ������ �������� ���������� ���� ������ ��� �����
������� ������� ����� ���� ������� ������� ���� ���
���� ������ ������� ����������� ���������� ���� ���
���������� �� �� ������� ����������� �� ���� GO ON TO THE NEXT PAGE.
����� ��� �� ��� ���������� �� ����� ����� ���� ����
�� ������� ��� �������� ����������� �� ��� �����������
������
��� ���� ������ ��� ���������� ������� ����� �������
�� ���� ������� ���� ��� ���� �� ����� ���� ���
������
��� �� �� ��������� �� ������ ���� ��� ��� ����������
���������� ����� ������� �� ���� ���� ���’�
�����
��� ���� �� ���� ��� �� ��� ������ ������ ������
�������� ���� ��� ���������� ���� ������ ���
����� �������
��� � ������������ ���� ��������� ��� ������ �� ����
���� ��� ���� �� ���� �� �������� �������
��� ���� ������� ������ �� ������������� ��� ����
����� ������� ���� ������ ��������� ������ ����
���� ��� ������������
74

1 1 -7-
1
��� ������� �� ��� ������������ �� ��������� ����� �� ��� ��� ��������� ������������� ������� �� ������� �������
�� ��� ��� ������� ��������� ������� ������ �� ��� ������ ��������� �������� ���� �� �������� ���
���� ��������� ���� ��������� ��������� “� ���� ������ ���� ���� �������� ������� ������� �� � ������
������ ������� �� �������� ��� �� ��������� ���� ������ ���� �������� ����� ��� ������ ���� ���������� ��
� ���������” �� ������� ����� ���� � �������� �� �� ���� ������� ��� �������� �� ��� ��� ������ �� �������
����������� ����� ��������� �������� ������� ���� ��������� ����� �� ���� �� ���������� ��������� ����
������ ���� ������ ����� ��� ��� ����� ���� ����� �� �� �� ��� ���� �� ������� �������� ���� ��� ���� ��
��� �� ����� ��� �� ������������ ���������� ���� � ���������� ����� ������� �� ��������
��� ������’� ����������� �� ����� ���� �������� ������� ����� ��� �� ��� ��������� ���� ���������� ��������� ���
����� ��� �� ��� ���������� ������� ���������� ����� �� ��� ���������
��� ������� �������� ���� � ��������� �� ��� ������ ��� ��� ��������� ������������� ������� �� ���������
������ ������� ����� �� ��� �� ����� ����� �� ���� ������ ��������� �������� ���� �������� ���
��� ������� ��� ���� �������� ����� �� ��� ���� ��� ������ ���� ����� �������� ������� ������� �� �
��� ������ �������� ����� �� ��� ��������� ������
��� ������� ����� ��� �������� ���� ��� ��������� �� ��� �� �� ������ �� ��� ��� ������ �� ������� ��������
���������� ��� ����� �� ��� ���� ��� ����� ����� ���� ��� ���� �� ���������� �������� ���� �� �� ��
��� ������� ��� ��� ��������� �� ��� �������� ��� ���� �� ������� �������� ���� ��� ���� ��
����������� ����� ����� �� ��� ��������� �� ��� ���������� ���� � ����� ������� �� ��������
��������� ��� �� �� ���� �� ��� ��� ������ �� ������� ����
��� ������� ���� ��� ������ ������� ���� ����������� �� ��������� ������� ���� �������� �� ���� ��
� ��������� ����� �� �� ��� ������� ��������� ���������� ������� �� � ������
��� ���������� ��� ������ ���� ������ ���������
��� �������������� ��� ����������� � ����� ���� ����� ��� ���� �������� ���� ������� ������� �� � ����� �����
�������� ��� �������� ������ ����������� ���� ��������� ��� ������ ���� ��������� �� ����� ���������
�������� �������� ��� ����� �� ������� ������ ��� ������� ��� ������ ��������� �� ������� �� ���� �������� ��
����� ���� �������� ���� ����� ��� ������ ��� ��� ������� ���� ���� ��� ���������� �� ������� �� � ������
��� ���� ��� ����� ����� ��� ���� ��� ��� ����� �����
��� ���������� �� ��������� �������� ����� �� �� ����������� ����
��� ���������� ����� ����� ������� ��������� �� �����
�� ��������� ���� ����� ���������� �� ��������� ����������
��� �� ��� ��������� �� ��������
��� ��� ���� �� ���� � ��������� �� ������ �� ��������
��� �� � ������� ��� � �������� ���� ��� � ������ ���� �� ������������ �� ��� ������� ����� ���� ���� ���� � ���������
������� ���� ������ ������� �� ���� �������� ������� ��������� �� ���������
��� �� � ������� ��� ���������� ���� � ������ ���� �� ���������� ���� ����� ������ �� ������� ����� �����������
��� ��� ���� � �������� ���� ��� ���� ������ ��� ��������� ���������� ���� ������� ���� ����
��� �� � ������� ������� ������ ��� ������� ���� �� ���������� ��� �� ��� ������� ��� ����� ������ ����� ��
��� �������� ����� ��� ������ ��� ������� ������ ����� ������� ��������� ���� ��������� �������
��� �� � ������� ��� ��� ����� � ������ ���� �� ���
����� ��� �� ��� ��������� ���� ���������� ��������� ���
���������� ���� ���� ������
���� ������ �� ��� �������� �� ��� ����� ���� ��� ���� ��
��� �� � ������� ������� � ������ ���� �� ��� � ��������
� ��������� �������� ����� ��������� ���� ����� ���������� ��
���� ��� ���� ������
��������� ��������� �� ������ �� �������� ������������ �� ���
�������
��� �� �� �� ����������� ����� ���� �� ���� �� �������
������� ��� ��� ��������’� �����������
��� �� �� �� ����������� ����� ���� �� ���� ��
������� ������� ����� ���� �� ���� �������� ���
��������’� �����������
��� �� �� � ����� ��� ����� ��� �������� ��������
���� �������� ��� ����� �� ���� �� ���� ��
������� ��� ��������’� �����������
��� �� �� � ����� ���� ������ �� ��� ��������’�
�����������
��� �� �� � ����� ���� �������� ������������
���������� ����������� ��� ��� ��������’�
�����������

GO ON TO THE NEXT PAGE.


74

1 -8-
1 1
��� ����������� 3ALMONELLA �������� ��� � ����� ����� �� ��� ��� ��� ������� ����� �� ��� ������ ��� �����������
������� �� ������ ��� ������� �������� ����� �� ��� ������� ������� �� ��� ��� ��� ��� ��������� ���
������ ���� ��������� � ��� ��������� ��������� ������� ����������� ��� ������� �������� �������� ���
� ����� ��������� �� 3ALMONELLA ��������� ���� ���� �������� ����������� ���� ��� ��� ������ �� ���
��� ��������� ������� �������� ��� ���� ����� ��� ���������� �� � ������ ���� �� ����� ����� ������ ����
��������� ��� ������������ ��� ������� ������ ��� ������� ���� ���� �������� �� ��������� ���� ��������
������ �������������� �� � ������� �� �������� ���� ���������� ��� �������� �� ���� ���� ��� ������ �� ���� ��
��� ��������� ������� �������� ��� ������� ������������ ����� ��� ������ ���
�������� ���������� �� ���������� �������� ������ �����
����� ��� �� ��� ���������� �� ����� ���� ����� ��
�����������
������� ��� �������������� �� �������� ��� ���� ����� ���
���������� ����� ��� �� ��� ���������� �� ����� ����� ����
���������� ��� ���������
��� ��� ��� ��������� ����� ������� ����� ��
����������� ��� �������� �������� ����� ���� ���� �� ��� �����
��� ������ �� 3ALMONELLA �������� �� ����� ������ ��� ��������� �� ���� ��� ������ ��� ��� �����
��������� ��� ���� �� ����� ����� ����� ���� �������� ������ � ����� ������������
��� ���� ������ ������� ���������� �� ���� ������� �������
�������� �� ��� ���� ���� ����� ���������� ��� ��� ������ ��� ���� ��� ������ ��� ����������
��� ��� ��������� ������ ����������� � ������ ������ ������� ��� ����� ����� ��� ���� �� ����
��������� �� ������� ���� ��������� �� �������� ������� ��������� ������
����� ���� 3ALMONELLA ���� ��� ������� ������� ��� ��� ��������� ���� ��� ��������� ��� ������ ���
��� ��� �������� ����� �� ��� ������� ������ ���� ����� �� ���� ��� ������ ������ ��� ��� ���
����������� ����� ����� ������ �� ��������� �� ��� ����������
3ALMONELLA ��������� ��� ��� ����������� ���� �� ��������� ����
��� ������ ��� ��� ����� ����� ���� ������
��� �������� �� � ����������� ��������� ��������� �������� ���������� ������ ��� ��� ��� ��� ����������
���������� ����� ��� �������� �� �������� �� ��� ��� ��� ����� ����� �� ����� �� ���� ���������
������� �� ������� �� ��� �������� ��� ������ ����� ��������� ���� ��� ��� ��� ��� ��������� ����
���� ���� ���� ������������ ����� ��� ��������� �� ��� ������ ���
��������� �� � ����� ���������
����������� �� ����������� ��� �������� ��� �������� ���
������������� ��� ��� �������� ��� �������� ����
������� ���� ������ �� �������� �� ��������
������������ ��� �������� ���������� ����
������� ��������� ����� ��� ��������� �� ��������� GO ON TO THE NEXT PAGE.
�� � ���������
��� ����������’� ����� �� ��� �������’� �������� �� ����
���������� �� ��������� �� ��� ������� ���� ��� ����������
��� �������� ��� �� ��� ����� ����������� �� �����
��� �������’� �������� �������
��� ����� ��� ������� ���� �������� ������� ���� ���
��������� �� ����������� ��� ���� ��������� ��
����� �������� �����������
��� ����� �� �������� ��� ����������� ���� ����
��������������� �� ��������� ����� ���� ���������
��� ����������
��� ������� � ��� ������� �� � ��������� ������ ��
��������� ���� ��� ������ �� ����� ��� �������
������� ��
��� ����� ��� ������� ���� ��� ���������� ���������
�� ����������� �� ������������ �������������� ��
��� ���������� ��������� �� �� ����� ���� �����
�������� �����������
74

1 1 -9-
1
��� ������� ����������� � ������ �� ���� ���� � ������� ��� �� ������������ ��� ������� ����� �������� ���� � ���
������ ����� ���� ����� ��� � ������� ��������� ���� ���� ����� ��� ��������� �� �� ������ ����� ����
�� ������� ����� ������ ��� ��������� ����� �� ����� �� ��� ��������� ��� ����� ���� ��������� ����� ���
����� � ����� � ����� ���� ����� ������ ��� ����� �������� �� ������ �������� ����� ��� ��������� �� ��
������������� ����� ���� ����� ���� ���� ����� ������ ��� ����� �� ����� ������ ������������ ��� ��� ����
����������� �� ��� ��������� ���� �� ����� �������� ����� �������� ���� �����
�������� ��� ����’� �������� ������� ��������
��� ������� �� ��������� �� ����� ��� �� ��� ���������
��� ��������� �� ��� ������� ����������’� �������� ��������� ���� ������� ��������� ���� �� ��� ��������
�� ���� ���������� �� ��������� �� ��� ������� ���� ��� ������
��������
��� �� � ������� �� ������ ���������� �� ���� ������
��� ����� �� ������� ��� ����������� ���� �� �������� ���� ���������� ���� � ��� ������������ ��
����������� ������� ��� ��������� �� ��� �� � ������� ������� �� ��� ��� ������������ ��
������� ������ ���� �������� ����������� �� ���� ���� ���� �� ������� ������ �� ������� �������
��������� �������� �� ���� ������������ ���� ����� ����
��� ����� �� �������� ���� ���� �� � ����� ������ ������� ��� ��� ��� �� ������� ������ �� ���
�������� ����������� �� ��� ���������� �� � ����� ������� ������� �������� �� ��� ��� ������������
����������� �� ��� ��� �� ��� ���� ������ �� �� ��� ������ ����������
��������� ��� ���������� �� ���� ���������� ��� �� � ������� ������� �� � ��� ������������ ����
��� ���������� ���� ��� ����� ���� � ������� ������� �� ���������� ������� ���� ����� �� ���
���������� ������ ��� ��� ����� ���� � ������� ������������ ��������� ������ ���� �������
��������� �� ���������� ��� ���� ���������� �� ������� �������� �� ����� ������������� ��� ��
������ ���� ��� ��������� ���� ������ � ������� ���� ��� ��������� ������ ��� �������
��� ����� ��� ������� ���� ����� ���� �� �� ���������� ������� �������� �� ��� ��� ������������ �� ���
����������� ������� ��� ��������� �� ������ ��� ������� �� ���
�� ����� ��������� �������� ����������� �� ��� ��� �� � ������� �� ���������� ���� � ��� ������������
����� �� ��������� ������� ���� ������� �������
��� ����� �� �������� ���� ���� �� � �������� �������� �������� �� ���� ������������ ��� ���� �� �� ������
����������� �� ��� ���������� ���� � ����� ���� �� ��� ���������� �� � ������� ���� ��� �����
����������� ���� ���������� ��� ���� ����� ���� �� � ��� ������������ �� ���� ��� ���������
�������� ������������ ������ ��� ������� ������� �������� �� ���
��� �� ��� ������������ �� � ��� ������� �����
��������� ������ ���� ������� ������� ��������
�� �� ������������ ���� ������� ������ ���
�� ���������� ���� ��� ���������� ����� ���
������������ �� ������� ���� ��� ������������
���� ������ �� ���� ������� �� �����
������������ ����� ��������� ��������� �������
������ �������� ��� �� ���������� ���� ���
�������������
��� �� � ��� ������� ����� ������ �� ������������
���� ������� ������ ��� �� ���������� ���� ���
�� � ��� ������� �� ����������� ��� ���� ���
�� ������� �� ����������� ��� �����������
���������� ������� ��� ������������ �� �������
���� ��� ������������ �� ������ �� ������� �����
����������� ������ �� �����������

GO ON TO THE NEXT PAGE.


74

1 -10-
1 1
��� �� ��� ���� ������ ��� �������� ���������� ������� ������� ��� � ����� �� ������ ��� �� ������������ ����� ���� ������’�
�������� ������������ ������������� ���� ���� ����������� ������������ ���� ����� ������� �� ��� �������� ����������
������ ��� �� ������� ����� ����������� ��� ���� �� ��� ���� ��� ������ ���� ����� ������ ���� �� ���� ��
����� ��� ����� ������ �� �������� ���� �� ������ ��������� ������������� �� ������ ���� ����� ���� �������� ������
��� �������� ���� ��������� �������� �� ��� ������ �� ��� ��� ����� ����� ���� ������’� ������������ ���� �����
�������� ���������� ����� �������� ��� ���������� ������ ������� ������� ������� �� ��� ��������� ����� �������
���� �������� ��� ����’� ��� ��������� ������� ���� ����� �� ����� ����������
��� ���������� �� ��� �������� ������� ��������� �� ����� ��� ���������� ������ �� ����� ���� �������� �������
��� �� ��� ��������� �� �������� ����� ��� �� ��� ��������� �����������
��� �� ��� ������ �� �������� �� ��� ���� ���� ��� ��� ������ ���� ���� ������� ��� ������������ ����
��������� ���� ��� ����’� ��� �������� ���� ��� ��������� ���� ����� ������� ���� ��� ������ ��
��������� ��� ������ ������
��� �� ��� ������ �� �������� �� ��� ���� ���������� ��� ����� ������ ��� ��������� ���� ��������� ����
���� ��� ������ �� ����� ����� �� �������� ���� ����� ������� ���� ��� ������� �������
��������� ��� ������������ ���� ������ �� �������� ����������
��� ��� ����’� ��� �������� ���� ��� �������� ������ ���� �������������
��� ���������� ������ �� ������ ��� �� �������� ������’� ������ ������ ���� ������
��� ������ ��� ��� ��� ������� �������� �� ��� ������ �� ����� ����� �� ������������ ���� ���� �� �
���� ���� �������� �� ����� ��� ���� �� ��� ��� ������
���������� ������ �� ������ ��� �� �������� �� ��������’� ������� �� ��� ������
��� �� ������� �������� ������������ ������������� ���� �� ������� �������� ������’� ������ �� ������������
����� ����������� �� ��� ���������� ������� ����� ���� ����� ��� ��������
����������� ���� �� ������
��� ���������� ��� �������� ���� ���� ��������� ������
��� �� � ������ ����� �� ������� ���� ���� ������ ���� �� ���� ���� ��� ���������� ������� �� ������ ��� �������
��� ����� � ������� ���� �� “����” ����� ���� ������ ���� �� ��� �����’� ����� ��� ���� ����������
��� �������� ��� ��� ��� ����� � ����� ����� ��� ������� �� ���������� ��� ���� ���� �� ������� ��� ����
��� ����� �� ������ �� ���� ���� ����� ��� ���� ��� ���� ������ ��������� ��� ���� ���� ������ ��������
���������� ����� �� ������� ��� �������� ���� ����� ������ ���� ��� ��������� �� ��� ������ ��� �����
���� ���� ���� �� �������� �� ����� ������� ��������� ������ �� ������� �� ��� �������� �� ���������
�� ����������� ��������� ���������� ��� ���� ��
����� ��� �� ��� ��������� ����� �� ���� ������ ��
������� ��������� �� � ������ ���������
���� �� ����� �� �������� ��� ���������
����� ��� �� ��� ���������� �� ����� ���� ������� ���
��� ���� ���� ��� ���� ���������� �� ���������
���������’� ���������
������� ������� ����� �� ��� ����� ���� ��������
�� ���� ��� �������� ��� ������� ���� ���� ���������� ���� ��� ������ ��
��� �� ����� � ������� �� ��������� �� ������� ��� �����������
�������� ���� ���� ���� �� ��� ����� ��� ������ ��������� ���� ������� ���� ����� ���
��� ���� ���� ��� �������� ������ �� ��� ����� ���� �����’� ����������
���� �� ���� ���� ��� ��� ������� ������ �� ����� ��� �� ����� ���� ���� �������� �� ����� ���
������� ��������� �� ������ ����������
��� ���� ����� ��� ����� �� ����� ��� ��� ��� ��� ��� ������� ������� �� �������� ������ ���� ������ ���
����� � ������ ������ ���� �������� �� ���� �����’� ������
��� �������� ��� ���� ������ �������� ���� ������ ���� ���
��� ��� ���� ����������� ���� �������� ������ ���������� ������� �� �������
��� ���������� ���� ����� �� ������� ���
��������

GO ON TO THE NEXT PAGE.


74

1 1 -11-
1
��� ��� ������ �������� �� �� �������� ������� �� �������� ��� �������� �� � ������ ��� �� �������� �� ��������� �� �
������ �� �������� ������ ��������� ���� �� ������� �������������� ���� ���� ������������� ����
�������� �� ������ ��� �������� �� ��� ���������� �� ����� ��� ����� ���� �� ������� ������� ���������
��������� ������ ��� ������ �� ��������� �� ����� ��� ��� ����� ���� ������ ��� ������� �� ��������������
�� ��� �������� �� ������ ��� �������� �� ����� �� ��� ������� ���������� ��� ������� ��������� ��
�������� �� � ������� ������� ���� ���� ���� �� �������� �� ��������� ��
���� ��������������
����� ��� �� ��� ��������� ��� �� �������� �������� ����
��� ���������� ������ ��� �������’� �������� �� ���� ������� �� ��� ������
������� �� ��������� �� ����� ��� �� ��� ����������
��� �� ��� �� ��������� �� ��������� ��� ��������� ����
��� �������� ��� ��� ���� �� �������� ������ ��� ���������� ������� ����� �� ����� ��� ���� �������
��������� ����� ���� ���� ����� ��� ��������� �� �������� ��� ���
��� �� ���’� ��� ��� ������� �� ������ �� ���� �� ����� ����� ���� ������� ����� ������� �� ���
��� ��������� ���� �� �� ��������� �� ��������� ����� ��� ��������� �� ���������
��� �������� �� ��� ���������� �� ����� ��������� ��� ����������� ��� �� �������� ������ ���� �������
��� ������� ����� ���� ��� �� ������� �������� �� ������ ��� ����� ����� ������� �� ������� ��� ���
������ ��� �������� �� ��� ���������� �� ����� ������������ ���� �������� ����� ��� �� ��������
��������� ������ ���� ������� ������ ��� �����
��� �� ��� �� ��������� �� ��������� ��� �������� �� ��� ������ ���� ���� ��� ����� ���� ��� �����������
��� ���������� �� ����� ��������� ���� �� ��� ���������� ������� ��� ��� ��� �� ��� �������
������� �� ��� ��������� ���������� ������ ����� ������� �� ���������� ���
��� ������ �� ��� ��� ������� �� ��� �������� �� ��������� ��� ������ �� ����� ����� ����
��� �������� ��� ��� ���� �� �������� ������ ��� ���� �������� ������ ��� ��������� �� ���������
��������� ����� ��������� ���� ������ ���� ������� �� ������
����� ����������� ��� ������ ������� �� ��������
���������� ���� ����� ���� ����� ����� ��� ���
�����
��� ������� ���� ���� ������� ���� �� ���� �����
����� ���� ����� ���� ��� ����� ������ ����
�������������� ���� ����� ���� ��� ���������
������ �� �� ���� ����� �� ���������� ��������

S T O P
IF YOU FINISH BEFORE TIME IS CALLED, YOU MAY CHECK YOUR WORK ON THIS SECTION ONLY.
DO NOT WORK ON ANY OTHER SECTION IN THE TEST.
74

2
  22 2 2
-12-

SECTION II
Time—35 minutes
23 Questions

Directions: Each group of questions in this section is based on a set of conditions. In answering some of the questions, it may be
useful to draw a rough diagram. Choose the response that most accurately and completely answers each question and blacken
the corresponding space on your answer sheet.

��������� �–� �� ����� ��� �� ��� ��������� �� �� ���������� �������� ��


����� ���� ����� �� �����
� ������� �� ����� �� � ���������� ����—���������� ��������
��� ���������� �������������� ������������ ����������
������� �������������� ������������ ���������� ���������� ������
���������� �������� ������
��� �������� ���� ������ �������� ������� ��� ����� ���
��� �������������� ���������� �������� �������
��������� ������������ ������
���������� ������������ ���������
��� ��������� ���� ��� ������� ��� ������ �����
��� ���������� ���������� ������������ ��������������
��� ������������� �������� � ���� �� ���� ���� ������ ���
�������� ������� ���������
�������� ������ �����
��� �������� ������� ���������� ����������
��� �������� ������ �������� � ���� �� ���� ���� ����� ���
������������ ���������� �������������
��������� ���� ��� �� ���� ���� ������ ��� ��������� �����
��� ���������� ���������� �������� ������� ��������������
��� ����������� �������� � ���� �� ���� ���� ����� ������
������������ ���������
��� ������������� ���� �� ��� ��������� ����� ��� ��� �����

GO ON TO THE NEXT PAGE.


74

2
  22 2 2
-13-

�� �� ��� ������������� �������� � ���� �� ���� ���� ������ �� ����� ��� �� ��� ��������� ������ ������� ��� �����
��� ����������� ����� ���� ����� ��� �� ��� ��������� �����
���� �� �����
��� ���������
��� ��� ������������� �������� ��� ����� ����� ��� �������� ������
��� ��� ������������� �������� ��� ������ ����� ��� �����������
��� ��� ��������� �������� � ���� �� ���� ���� ������ ��� ���������
��� ����������� ����� ��� ���������
��� ��� ������������� �������� � ���� �� ���� ����
������ ��� ��������� ����� �� �� ��� ��������� �������� ��� ������ ����� ���� ���� �� ���
��� ��� ����������� �������� � ���� �� ���� ���� ��������� ���� �� ���� �������
������ ��� �������� ������ �����
��� ��� ������������� �������� � ���� �� ���� ����
������ ��� ��������� �����
�� ���� �� ��� ��������� ���� �� ����� �������
��� ��� ��������� �������� � ���� �� ���� ����
��� ��� �������� ������ �������� ��� ����� ����� ������ ��� ��������� �����
��� ��� ��������� �������� ��� ������ ����� ��� ��� ��������� �������� � ���� �� ���� ����
��� ��� ��������� �������� � ���� �� ���� ���� ������ ������ ��� ��������� �����
��� ����������� ����� ��� ��� ����������� �������� � ���� �� ���� ����
��� ��� ��������� �������� � ���� �� ���� ���� ������ ������ ��� ��������� �����
��� ������������� ����� ��� ��� ��������� �������� � ���� �� ���� ����
��� ��� �������� ������ �������� � ���� �� ���� ������ ��� ����������� �����
���� ������ ��� ����������� �����

GO ON TO THE NEXT PAGE.


74

2
  22 2 2
-14-

��������� �–�� �� ����� ��� �� ��� ��������� �� �� ���������� �������� ��


��� ��������� ���� ����� �� �������
���� ��� ����������—������� ������� ������� ��� �����—����
��� ������� ����������� ������� ������������
���� � ������ �� ���� ������ ��������� ���� ������� �� � ���������
������� ��� ���������� ������ �����������
�����—������������ ��� ���������� ����������� ��� ������������
��� ������� ������������ ������ ��� ����������
��� �������� ���� �� ����� ��� �� � ����� ���� ���� ��� ���������
������� ����������� ������� �����������
������ � ������� �� � ��������� ��� �� ��� ������� ��� ��������
��� ������� ����������� ������� ������������
�� ��� �������� �� ������� �� ��� ��������� ������������
������� ��� ���������� ������ �����������
��� ��� ��������� ������� ��� ��� ����������� ������� ����
��� ������� ��� ���������� ������ ������������
���� �� ������� ���� ��� ����������� ��������
������� ������������ ������� ����������
������’� ������� ���� �� ������� ���� ��� ��� ���������
��� ������� ����������� ������� ������������
��������
������ ��� ���������� ������� �����������
������’� ������� ���� �� ������� ���� ���� ������’� �������
��� �����’� ��������

GO ON TO THE NEXT PAGE.


74

2
  22 2 2
-15-

�� ����� ��� �� ��� ��������� ���� �� ����� �� ����� ��� �� ��� ��������� ������ �� �����
��� ������’� ������� �� ������� ���� ��� ���������� ��� ������ ����� ��� ����������� ��������
�������� ��� ������ ����� ��� ���������� ��������
��� ������’� ������� �� ������� ���� ��� ����������� ��� ������ ����� ��� ����������� ��������
�������� ��� ������ ����� ��� ��� ��������� ��������
��� ��� ���������� ������� �� ������� ���� ������’� ��� ����� ����� ��� ����������� ��������
��������
��� ��� ���������� ������� �� ������� ���� �����’� ��� �� ������ ����� ��� ���������� �������� ����� ��� �� ���
�������� ��������� ����� �� �����
��� ��� ����������� ������� �� ������� ���� ������’�
��� ��� ����������� ������� �� ������
��������
��� ��� ��� ��������� ������� �� ������
��� ��� ���������� ������� �� ������
�� �� ��� ����������� ������� �� ������ ����� ��� �� ���
��� ��� ���������� ������� �� �������
��������� ����� �� �����
��� ��� ����������� ������� �� �������
��� ������ ����� ��� ����������� ��������
��� ������ ����� ��� ��� ��������� ��������
��� ������ ����� ��� ���������� ��������
��� ������ ����� ��� ���������� ��������
��� ����� ����� ��� ����������� ��������
GO ON TO THE NEXT PAGE.
74

2
  22 2 2
-16-

��������� ��–�� ��� ����� ��� �� ��� ��������� ����� �� ��� ������ �� ���
����� �����
����� ���� ���� �� ����� ��� �� ������� ������� ��� ������ ��
��� ������ �����
������ ��� ���������—������� ������ ������ ���������� ������
��������� �����
��� ������—������� ���� �� ����� ���� �� ���� �� ����� ���
������ ������ ��� �����
����� ���� ����� ���� �� ���� ���� �� ���� �� ���� ��� �� ���
��� ������ �����
����� ��� ���� ��� ������ �����—����� �� � ������ �����—��
��������� �����
������������� ��� ���� ���� �� ����� ��������� �� ���
������ ������ ��� ������
��������� ������
��� ����� �����
�� ��� ��� �� ����� ����� �� ����� ��� ����� ������ ��� ����
��������� �����
�����
������� ������ ��� �����
�� ��� ��� �� ����� ����� �� ����� ����� �� ���� �����
��� ������ �����
������ ��� ��������� ��� ��� ���� �������� �� � ����
������ ��� ����������
����� ��� ��������� ��� ��� ���� �������� �� � ����
����� ��� �����
����� ��� ������ ��� ��� ���� �������� �� � ����
��� ������ �����
����� ��� ������
��������� ��� �����

GO ON TO THE NEXT PAGE.


74

2
  22 2 2
-17-

��� ����� ��� �� ��� ��������� ���� �� ����� ��� �� ����� ��� ������� ��� ����� ����� ���� ��� ������ �� �����
��� ���� ������ ��
��� ����� ��� �� ������������ ���� �� ����� ������ ��
����� ��� ������ ��� �����
��� ����� ��� �� ������������ ���� �� ����� ��� ������ ��� ������
��������� �� ����� ��� ����� ��� ���������
��� ����� �� ���� �� ��� �� ��� ����� ��� ����� ��� ������
��� ��������� �� ���� �� ��� �� ��� ����� ��� ��������� ��� ������
��� ������ �� ���� �� ��� �� ��� �����
��� �� ������ ��� ����� ��� ���� �������� �� � ���� ����� ���
��� �� ��� �� ��� ���� �� ����� ������ ����� ��� �� ��� �� ��� ��������� ����� �� �����
��������� ���� �� �����
��� ����� �� ������� ��� ����� ����
��� ��� �� ��� ���� �� ����� ������� ��� ����� �� ��� ���� �� ��� �� ��� �����
��� ��� �� ��� ���� �� ����� ���������� ��� ������ �� ��� ���� �� ��� �� ��� �����
��� ��� �� ��� ���� �� ����� ������� ��� ��������� ��� ����� ��� ���� �������� �� � ����
��� ������ ��� ����� ��� ���� �������� �� � ���� ��� ��������� ��� ������ ��� ���� �������� �� � ����
��� ����� ��� ������ ��� ���� �������� �� � ����
��� �� ��� �� ��� ���� �� ����� ������� ���� ��� �� ���
��������� ����� �� ���� �������
��� ����� �� ������� ��� ����� ����� ����
��� ��� �� ��� ���� �� ����� �������
��� ��������� �� ��� ���� �� ��� �� ��� �����
��� ������ ��� ����� ��� ���� �������� �� � ����
��� ����� ��� ����� ��� ���� �������� �� � ����

GO ON TO THE NEXT PAGE.


74

2
  22 2 2
-18-

��������� ��–�� ��� ����� ��� �� ��� ��������� �� �� ���������� ����������


�� ������������� �� ��� ��� ���������� �����������
��� ������� �� � ����������� �������� ���� ������ �� �����
��� ����� ����������� ��������� ���
��� ������������� �� ���� �� ��� ���������� ����������—
������ ����������� ������ �������� ����
��� �� ����� ���������� ��� ��� ����� �� ������ �����������
��� ����� ����������� ��������� ����
������� ��� ������������� ��� ���������—������ ���������
������ ����������� �������� ���
�������� �������� ���� ��� ����—��� ��� ��� ���� �� ��
��� ����� ����������� ������ ��������� �������
��������� �� ������������ ��� �� �������� �� ���� �����������
������ ����������� �������� ���� ����
��� ��������� ����������� ������
��� ����� ����������� ������ �������� ����
����� ���� �� �������� �������� ���� ������� �� ��� �� ���
������ ����������� ��������� ���
���������� �����������
��� ����� ����������� ������ �������� ����
�� ��� ��� ���� ��� ���� ��������� �� ���� �� �� ���������
������ ����������� ��������� �������� ���
�����������
�� �������� �� �������� �� ��� ����� ���������� ���������
���� ��� ���� �� �������� �� ��� ������ ����������
���������
�� ������� �� ��� �������� �� ��� ������ ����������
��������� ���� ���� ������� ��� ���� ���� �� ��������
GO ON TO THE NEXT PAGE.
�� ���
74

2
  22 2 2
-19-

��� �� ������� �� �������� �� ��� ���� ���������� �������� ��� �� ������� ���� �� ��� ������������� ��� �������� �� ���
�� ���� ���� ����� ��� �� ��� ��������� ���� �� ����� ���������� ����������� ���� ����� ��� �� ��� ���������
���� �� �������� �� ��� ����� ���������� ���������
��� ����� �� �������� �� ��� ������ ����������
��������� ��� �����
��� �������� �� �������� �� ��� ����� ���������� ��� ��������
��������� ��� �������
��� �������� �� �������� �� ������� ���������� ��� ���
��������� ��� ����
��� ������� �� �������� �� ��� ������ ����������
��������� ��� ����� ��� �� ��� ��������� ������ �� ��� ��������
��� ��� �� �������� �� ��� ������ ���������� ���������� �� ������������� �� ��� ������ ����������
��������� ���������
��� ������ ��������� �������� ����
��� ����� ��� �� ��� ��������� ����� �� ��� ��������
��� ������ �������� �������� ����
���������� �� ������������� �� ��� ����� ����������
��� ��������� �������� ���
���������
��� ��������� �������� ����
��� ������ ��������� �������� ������� ��� �������� ����
��� ������ ��������� �������
��� ��������� ������� ��� ����� ��� �� ��� ���������� �� ����������� ��� ���
��� �������� ��� ���������� ���� �� ������� �� ��� �������� �� ��� ������
��� �������� ���� ���������� ��������� ���� ���� ������� ��� ����
���� �� �������� �� ��� ����� ���� ��� ���� ������ ��
��� ����� ��� �� ��� ��������� �� � �������� ��� �������� ����������� ��� ���������� �� ������������� �� ���
���� �� ��� �� ��� ������������� ��� ���� �� ��������� ���������� �����������
��� ������ ������� ��� �� ������� �� �������� �� ��� ����� ����������
��� ������ �������� ������� ��������� ���� ������� ��� ���� ������ ����
��� ������ �������� �������� ��� �� �������� �� ���� ���������
��� ������ ��������� ������� ��� �� ������� �� �������� �� ��� ����� ����������
��� ������ ��������� �������� ���� ��������� ���� ��� ���� ���� �� �������� �� ����
���������
��� ������ ������� �� �������� �� ��� ������
���������� ��������� ���� ����� ��� ���� ����
�� �������� �� ���� ���������
��� ������ ������� �� �������� �� ��� ������
���������� ��������� ������� ������ ��
�������� �� ��� ���� �������� �� ����
��� ������ ������ ������� �� ���� �� �������� �� ���
������ ���������� ��������� ������� ���� ��
�������� �� ���� ���������

S T O P
IF YOU FINISH BEFORE TIME IS CALLED, YOU MAY CHECK YOUR WORK ON THIS SECTION ONLY.
DO NOT WORK ON ANY OTHER SECTION IN THE TEST.
74

3 -20-
3 3 3
3 3
SECTION III
Time—35 minutes
27 Questions

Directions: Each set of questions in this section is based on a single passage or a pair of passages. The questions are to be
answered on the basis of what is stated or implied in the passage or pair of passages. For some of the questions, more than one
of the choices could conceivably answer the question. However, you are to choose the best answer; that is, the response that
most accurately and completely answers the question, and blacken the corresponding space on your answer sheet.

����� ��� ������ �� ���� ��� ������ ���� ��������� ���� �� ��� ���� ����� ��������� ���� �� ��� ���� ��
����������� �������� ��������� ��� ������� ����� �� ����� ������� ���� ������� ����� ��������� ���’�
����������� ��������� �� ����� �� �������� �� ��� ������ ��� ����������� ������������� ���� ������� ��
���������� �� ����� ��� ���� �� ��� ��� �������� �� ���� ��� ���������� ��������� ���� ��� �������������
��� ����� ���� �� ��� ���� ������ �� ��������� ����� �� ��� �������� ������� ��� ���������� �� ��������� ���
���� ��������� ������� ����� ����������� ���� �� ������ ������ ����������
��� �� ����� ������� ��� ����� ���� ����������
����� ��� ������������� ��� ���� ������� ����� ���� �� ����� ��� �� ��� ��������� ���� ���������� ��������� ���
�������� �� ����������� ������������� ��� ������ � ���� ���� ����� �� ��� ��������
���� ��� �� ���� � ���������� ��������� ��� ��������� �����
��� ������� ����� ������� �� ������� ������������ ����
� ���� �� ��� ��������� ������� �� ������ ��� ����
������� ��� ������ �������� ����� �������� ������
*OY 0ARFUM� ��� ���� ����������� �� ����� ������
��� � ����������� ������� ������ ��������� ����
��� ���� ��� ��������� ������� ���� ����� �� ��
��� �� ���� �������� ���� ����� ������ �� �
�������� �� ������ ���� ��� ����������� ��������
����������� �� ����� �� ���������
���� ������� ������� ���� ����� ����� ��������� ��������
��� ��� ��������� ������ �� ��� ������� �������� ��
���� ����� ���� �� ���� ��� ������� ���� �� ���
��� ������ ���� �� ��� ����� ����� �������� ���
�������� ��������� ���������� ������� ��� ���������
��� ���������
�������� ���������� ��� ��� ������� �������� ���
��� ����� �������� ��� ����� �� ��� ��� �������
����� ������ ��� ����� ��������� ��� ������� ������ ��
������� ��� ��������� �� �����
���� � ����������� ������ ��� ������� ����� ��
��� �������������� ��� ��� �������� ������ ��
�������������� ������ �������� �� �� ��� ��������
�������� �� ������ ����� ���� �� ����� ������� ���
���������� ����� ���� ���� ���� ������ �� ����
������� ����������� �� ������� �������������� ��
���� ������� ����� �� ��������� ����� ��� ���������
������������
��������� �� ������� �� ����� ��� ��� ������� ���� �
���� ������� �� ����� ������� ���� ����� �� ���� ��� �
�� �� ����� ��� �� ��� ��������� ������������� ����� ���
������� �� ����� ��� ���������� ������� ���� �����
������ �� ��� ������� �� ���� ������ �� ������� ���� �
������� ��� �������� �� ��� ����� �� �� ������
������� ������������ �� ��������� �� �������� ��� �������
��������� ���� ������������
�� � ������� ��������
��� ���� �������� “�����” ���������� ����
���� ������� �������������� �� ��������� �������� ��� ��� ��� ���������� ����� ��� ������� ���� �������
������� �� ��������� ����������� ���� ����� �������� *OY 0ARFUM�
����������� �� ����� ����������� ������ ������� ��� ��� ���������� �� ���� �� ������� �� ����������
����������� �� ������ �� ��� ���� ��� ��� ����� ���� �� ��������� ���� �������
������������ ������ ��� ������� �� �������� �� �� ��� ��� ���������� �������� � ��������� ��������
���� ������ �� ��� ��������� ��� ��� ������������ ��������� �������� ���� � ������� �������� ���������
�� ��������� ������ ���� �������� ������ ������� ������ ��� ��� ���������� �� ���� �� ���� ��� �������
�� ��� ������ �� ��� ����� � ��������� �������� ��� ������� ���� � ����� ������� �� ����������
���� ������ �� ��������� ����� �� ���� ��������� �� ��� ��� ���������� ����� � ���������� ������� �������
��������� ���� ���� �� � ����� �������� �� �������� ��� ������ �� ��� �������’� �������� ��������
���� �� ������� �� ��� �������� �� �������� ����� ����
������ �� ��������� �� �� � ��������������
����������� ��������� ��� �� ��� ���� �������� ��
��� ������ ��� ������� ��� �������� �� ���� �� �������
������� ��� ������ ���� ����� ����� ������ ��� ��
���� ����� ����������� �� ���� �������� ��� ����� ���� ��� GO ON TO THE NEXT PAGE.
����������� ����� ��� ��� ������������ �������� ���� ��
� ��� ����� ������������� ��� ������� ���� �������� ��
����� ��� ������ �� ��� �������� �� ������ ���� ���
��������� ��������� �� ��� ������������ �� �����
���� �������� ��������� ���� ������������� ��������
������ ������ ����������� �� �� ������ �� �������� ��������
74

3 3 3 3 -21-
3
�� ��� ���� “�����” ����� ��� ������ �� �� ��� ���� ��������� ���� �������� �������� ����� ��� ��
��� ��������� �����������
��� ���������
��� ������� ���������� ��� ��� ����� �� ��� �����’� ���� �������� ��� ���
��� ���������� �������� ����� �� ���� ����������
��� ������ ���� ��������� �� ��������� �������� ��� ��� ������������� �� � ���������� ������� �� ��� �
��� ������ ���� ��������� �� ������� �������� ���� ��������� �� ��� ��������
��� ��������� ���� ���� ������� ��� ������ ���������
�� ��� ������� �������� ��� ���� ������� ��� ����� ��� �� �� ���� ����� ��������� �����
��� ��������� ���������� ����� ���� ��� ������� ������ �� ��� ���� ����� ����� ������
���� ����������� ���������
��� � ���� �� ��� ��� ����� ����� �� ���������
��� ��������� ���� ���� ������� ���� ���� �� �����
���������� ������� ��� �������� ���� �� �������
������� �� �������� �� ��� ����� ��������� �����
��� �� ��� ���� �� ���� ��� ��� ������ ��� ����������
�������
����������� �� ���� ����� �����������
��� � ���� �� ��� ���� ���������� ���� �� �� �� �������
�� ����� ��� �� ��� ��������� ���� ���������� ��������� ���
��� ��� ���� �� ���������� ��������
������������ �� ��� ��������
��� ��� ���� ����� �� ��� ������� ���� ����
��� ���� ����� �� ��� ��� �������� �� ������� ��� ��� ����� ��������� ����� �� ������������ ���
������ ���������� ��������� �� ���� �����������
�� ��� ������ ����� �� ���� ������ �� ���� ����� ��� �� ��� ����� ����������� ��� �������� ����������� ���
��������� �������� ����� *OY 0ARFUM �� ����� ������ ��� ��� ��� ����� ��������� �������� �� �����������
������������
��� �� ���� ���� ��� ��� �������� �� ����������� ����
��� ��� ����� ��������� �������� � ������� ��� ������
��� �����
���������� ������� � ���� ��� ���� ������� ���
��� �� � ���� �� ���� �� �� �� ���� ��������� ���� �
��� ����� ��������� ��������� ��� ������� ���
����� ����� �� ����������
���������� ��������� �� ���� �������
��� �� ��� ��� �������� �������������� �� ��� ���� ��
��� ��� ����� ��������� ���� ��� � ��������� ��
������� �������
�������� ������� ��� ������ ���������� �������
��� �� �� � ��������� ���� �� ����������� ���� �� ������
� �������� �� ���� ���������� ��� ��� �����
���� ������� ������
��������� �������� � �������� ������� ����
��� ��� �������� ������� �� ������� �� ���� �����
�������� ��� ���������
�������� �� ��� ������
��� ��� ����� ��������� ����� � ��������� ��� ������
���������� ������� � ���� ���� ����� �� �������
�� ����� ��� �� ��� ��������� �� ���� ��������� �� ���� ���
��� ������ �� ���� ��������� ��� ��� �����
������ ����� ��� “������� ���� ��������” ����� ����
��������� �������� � �������� ������ �� ���
��� �� ��� ������ ������� ��� ������ �� ������� ���������
������ �� �������� ����� ��� �� ���������� ��� ��� ����� ��������� �������� � �������� ���
��� � ����� ������ ��������� ��� ������� ������ ���������� ������� ��� ������������ ��
����������� ���������� ������������ �� � ����’� ���� �������� ��� ��� ����� ��������� ��������
�������� �� �������� ��� ������� ���� ��� ����������� ��� ���
��� � �������� �� �� ��� ��������� ��� ���� ��� ������ ��������
������ ������� �� ����������� �� ���������
��������
��� � �������� ��������� ��� ��������� ��� �������
�� ������ �� ��� ������ ��� ��� ���� �� ���
��������
��� �� ��� ������ ���� ��� ������� ��� ������ �� ��� GO ON TO THE NEXT PAGE.
������� �� ����� �� �������� ��� ������ �� ��� ���
�������
74

3 -22-
3 3 3
3 3
“�������� �������” �� � ��������� �������� ���� �� ����� ��� �� ��� ��������� ���� ���������� ��������� ���
�������� �� � ������’� ��������� �������� ����������� ���� ����� �� ��� ��������
����� � ������ ������ ���� ����������� �� �������� ��
��� �������� ����� ��� ������ �� ��� ���������� ��
�������� �� �� �������� ������� ����� ����� �� �� �����
�������� �������� ��� ������������� �� ������
��� �� ��������� � �������� ���� �� ������� �� ���’�
������ ��� ���� ������������ ������� ��������
��������� �� ���� ����� ��� �� ��������� �� �����
��������� �� ������������� ��� ����� ���������
���� ������� ������� ���� �� ��� �������� �� ������ ��
��� ��� �������� ��������� ��������� ��������
�� �������� �� �������� ���������� �� ������ ��
�� �������� ������� ��� ���� ����������
������������ ���������� ��� ������� ���������� �����
������������ �� ��� ������� ����� �� �� ���
���� �� ���� ���������
���������� ������� ������’ ����������
�������� �� ��������� �������� ��� �������� ������
��� �������’ �������� ���� ������ �� ��� �����
��� ������������� �� �������� ������� �� ������ �������
�� �������� ������� �� ��������� �� �������
������� ��������� ��������� ����� ���������� ����
������������� ������������ �� ��� ���� ��������
��������� ���� ��� ��������� ��� �� ����� ���������� ��
��� ��������� �������
���� ����� ������ � ���������� ����� ����� �� �������������
��� ��� ����� �������� �� �������� ������� ���
�������’ �������� ���� ������ �� ��� ����� ��
�������� ��� �������� �������� ���� ���
�������� ������� �� ��� ���� ������������ �� �����
����������� �� �� �������� �� ��� �������
������������ ��������� �� �� ���� ��������� �� �������
��������� �� ��� �������� �� ��� �������
������������� ������������ �� ��� ��� ���������
��� �������� �������� �� ������� ��� ���������� ��
���� ������ ����� ��� ��� ������ ������������ ��������
�������� ������� ��� ���������� �������’ ������
����������� ����� ��� ������ �� ����� �� ������������
�� ��� ����� �� ��� ��������� ��� ��� ����������
������������� �������� �������� ���� ������ ���
��� ������� ����������� �� ��� ��������’�
������ ����������� ���� ������� �� �� ������� ����� ���
��������������
���� �������� ��� ������ �� �� ��������� �� ���� ��������
���� ��� ���� ��� �� ���� ����������� �������� �������
��� �� ��� �� ���� ���������� �������� ���� ��� ������� ����
��� ���� ������� ������ ���� �� ������� ��� ��������
����� ��� �� ��� ��������� �� �� ������� �� ��������
���������� �� ����������� ��� ���� �������� ���� ���
��������
��������� ���� ��� ���������� ����� �� �������� ��
������������� ������������ ��������� ���� ���� ��� ������� ������ ���� � ������ �� ��� �������� ����
���� ���������� ������ �� �� �������� ������� �� ��� � ������� �������� �� �������� ��� ������ ��
���������� ������ ���� ��������� �� ��� ���������� �������
�������� ������� ���������������� ����� �� ��� ��� ���������� �� ������� ���������� �� � �������
�������� ����� ��� ����� ������ �� � ���������� ������� ��������� ������������ ���� ���’� ������
������� �� �������� ���� ���� ��� ����� ������ �� ��� ��� �� ��� ���� ���� ������ �� ����������
���� ���������� ��� ������� ��� ���������� ��� ���� ��� ���������� �� ��� ����������’� ���������� ����
�������� �� ��� � ���������� ������� ���� ���� �������� ���’� ������ ��� � ����� �������� ���������� ��
�� ����� ��� ������������� �� ��� ������ ���� �� �� ���� ��������� ���� ���� �� ��� ����
�� ������� �� ��� ���������� � ����� �� �������� ��� �������� ��� �� ������ ������ ������� ����������
������� �� ���� ��� ����������� ��� ��� �������� �� ��� �������� ��������� �� ��� ����������’� ����
���� ���� ������� �� ������� ��� �� ���� �� ���� ������— ��� ��������� ���� ���’� ������� ����� �����������
�������� “��� �����” ����� ������ ���� �������� �� �� ������� �� ���������� ��� ���� ����������
���������� ������������ �� ������ ����� ���� ������ ������������� ������ �� ����������
���� �� �� ��� ���� �������� ���� �� ������� ����������
�������� �������� ������� ��� ���� ������� ��� ��� ����� ��� �� ��� ��������� ���� ��� ������ ������� �� �
���� ������ ��� ����� ��� �������� �� ��� �� ��� ��� ����� ������ ���� �� ���� ��������� �������� ����������� �� ���
��� �������� ��� ������������� ���� �� ����������� ������� �� �������� ��������
��������� ���� �� ��� ����� “����” �� ����������� ��� ������� ������ �� ��� ���������������� �������
�������� ������������ �������� �� ��� ��������� �� �� ������� ��� ����������’� ������� ������� ��
��������� ���� ���� ��� �������� ����������� ��� �� ���� � ����� ����
���� ������ ����������� ������� ��� ����� ��������� ���� ������ ��� ���� �������’ �������� ����� �� ��������� ������’
����������� ����� � ����� ��� ������� ����� �� �������� �������� ��������� �� � �������
����� �������� ��������� ��� ����� ���� ��� ��������� ��� ��� ����������� �� ���� �������’ ������� �� �������
�� �������� �� ��� ��� ����� �������� ������� �� ����� �� ������ ����� ����� ��� ���� ��������
����� ���������� �������� �� ��� ����� ������������ ��� ��� ������’ ������ �� ������ �� � ���� ���� ���������
���� ���� ���� �������� ����������� �� ��� ��� �� ��� ���������� ��� ���� ���� ��� �������
���� ����������� �� ���� ��������� �������� ������� ��� �������’ ������� ��������� �� ����������� ������
��� ������ �� ����� �������� ���������� ���� ����� ����� �� ��� ��������� �� ��������� �����������
� ��������� ��������� ��� ������� ��� ���� ������
���������� ����������� ������� ���� �������
GO ON TO THE NEXT PAGE.
74

3 3 3 3 -23-
3
��� ��� ������ ��������� ��� “��������� ���������” ���� ��� �� ������ ���� ������� ������� ���� ��������� ���� �� ����
������ ������ ����� ��� ��������� �� ������������� “��� ��������� ��� �� ����� ���������” �����
���� ��� ������ ���� ������ ����� ���� ����� ������� ����
��� �������� ���� �� ����� ���� ����������� ���������
����� �������� ���� ��� ��������� �� ��������
����� �� � ����� ��� ��������� ��� ��� ������
�������� ����������� ��������� ����� �� ��� ����� ��� �������� ������ �� ������ ��� ������� �� ����� �����
��� �������� ���� ������ ����� ���� ����� ���� ���� ��� ��������� ���� ��������� ���� ����� ��
������� ��������� ��� ������ ���� �� ����� �� ���� ��� ���� �� ��� �������� ����������� ����� ����
������ ����� �������� ������ ������ ���� ����� �������� �� ��� ����������
��� ������� ���� �������� �������� ���� �� ������ ��� �� � �������� ������ �� ��������� ��������� ��
���������� ����� �� � ����� ���� ���� � ������� ����������� � ��� �� ���������������� ���� ������
������ ���� �������� �������� ��������� ����� ���� �� ���� �� ��� �� ��������� ��� ����������’�
�� � ����� ���������� �������� �� ����������
��� �������� ���� �������� ������� �� ���� ���� �� ����� ��� ���������� ������� �� ����� ����� ������� �� �����
�� �������� �� � ����� ������ ��� ���������� ��� �� ��� ������� �� ����� ����� ��� ��������� ������
�� ����������� �� �������� ������’ �������� ���� �� ����� �� ������� �������� ����� �������� ��
��� ��������� ���� �������� ����������� �� ���� ����� �� �� �� ���� �������
�������� ���� ��������� ���� ���������� ������� ��� ������� ��������� �� ������� �� � ����� ��
���� ������� ����������� ���������� ��������� ������’ ��������� ��� ����
������� ���� �� ����� ���� ����������� ���� ����
��� ��� ������’� �������� ��������� �������� ������� ��� ���� ����� ��������� �� ��� ������’� �������
���������� �� ��������� �� ��� ���� ����� ���� ��� �������� ��� ���� �� ������
� �������� ���������� �� �� �� ������������
��� ��������� ���� ��� ��������� ��� ������ ��
������� ��� ����������’� ���������� �� ��� ����
������ ���� ������� ���� ���� �� ��������� ���
�����������
���������
��� ��������� ������ ��� ��� �� ������� ������ ���
��� ��� ������� ���� �������� ������� ���� ���� �������
������� ���������� �� � ���� ��� ��������� �� ���
������� ����� ��� �� ��� ��������� ���������� ���������
����� ���������
����� ������� �� ����� ��������
��� ��������� ���� �������� ������� ���������� ��
��� ���� ������� ��������� �������� ���������� ��� � ������ ������ �� ��������� ���� ��� �������
�������� �� ��� ��� ��� �������� ����������� ��� �� ����������
��� ��������� �� ��� ��� �� ��� ������� ���� ��� ������� ���������� �� ��� ����������� �� ����
������� �� �������������� ��������� ��� ��� �� ���������
��������������� ��������� ��� � ������ ������ ���� ���� �������� ����� �����
��� ��������� �� ��� ����������� ��� ��� �� ������� ������� ������� �� ��� ��� ������ ��� �������
������� ������� ���������� �� ��������� �������� ���� ���� �� ��� �������� ����� �� ���
���������� ���� ����� �� ��� ������
��� ��� �������� ������ �� ����� �� �������
��� ��� ������’� ���������������� �� �������� ������� �� ��� ������������� ���� ���������� ���� �������� ���
������� �� ����� �� ����� ������ �� ���� �������� ��������� �� ������ ��� ��������
��������
��� �������� ������� �� �������’ �������’ ��������� ��
��� ��� �������� ������ ������ �� ��� ��������
�������� ������� ��� ���������� �������� �����
�� �� ���� ��� ���������� ���� ��� �� ������ ��
�� ��������� ����� ����������
��������� ���� ���������� ��� ��������
��� ����������� ������� �� ������� ��� ����� �������
����������� �� ���������
��� ������������ �� �������’ ������� �� ������
��� ��� �������� ������ �� ����� �� ����� ������ �� �
��� ������� �� ������’ ��������� �� ����� �����
������’� ��������� �� �������� �������������
��� ��� ������� �� ������� ��������� ���������
�������� �������� ��� ����� �����������
��������� ����������
��� �������� ������������ �� �������� ���� ��
���������� ��������� �� �������� �������
��� ���������� ������� �� �������’ ���������
��������
��� �������� ���� ��� ��� �������� ��������� ����
GO ON TO THE NEXT PAGE.
����� ����������� ��� �������� �� ����� ��������
������������ �� ��������� ����� ����������
74

3 -24-
3 3 3
3 3
Passage A ��� ���� �������� ��� ��������� ���� ��������� ����� ���
�� � ��������������� ��� ��� ���� ������ ������� �� ��� ��������� ����������
������ ���� �������� ����� ���� ��� ���������� �� ����
��� ������ ������ �� �������� ��� ������� ���� ���
������ ���� ���������� �������� ���� ������ �������� ��
������� �� ����� ��������
������������� ��������� ������� ��� ��� �� �����
��� ���� ���������� �������� ���� ��� ����� ����
��� ����� ��� ���� ��� �������� ���� �� ��� ��� �� ���� ���
������������ ��������� ������� �� ��� �����
���� ������� �� ������� ��� ��� ��������� ��� ��� ���
��� ��� ������� ���� ��� ��� ���� �� �����������
���� ������� ��� �� ������� �������� ���� ������ �����
�������� �� ��� ������ �� �����������
������� ��� ����� ������ �� ��������� ������ �����
��� �� ��� ���� ���� ����������� �� � ����������
�������� ������� ������ ��� ������� ���������� �� ��
������������� ��� ���������� ���������� ���� ���
���� ������ � ���� ��� �������� ����� ������ ��� ������ ��
�������� �� �������������
�������� ����������—� ����� ���� ����� ���� ����
��� ��� �� ������ �� ���� �� ������� ���� ����������
���� � ������ ���������� ���� �������� �� ���������
������ ��� ����� �� ������� ���
������� �� ��� ����� ������ ���� �� ��� ������ �� ���
����� ���������� ���� ������������ ������� ��������
��� ����� ��� �� ��� ��������� �������� ����� � ���� �� ���
���� ���� ������� �������� ���� ��� ���������������� ������
�������� �� ������� � ��� ��� �� ���� �� ������� ��
������ ������� ��� ���� �� �����������—��� ���� ����
��� ������ ������ �� �������� ������� �� ����� ������ ��� ������ ��������
������ ������� ����—����� �� ��� �������� �� � ��� ���� ������
������������� �� ����������� �������� ��� ��� ������ ��� ���������
���� ����� �� ��������� ������ ������ �� ���� ������ ���� ��� �������������
����� ���� ������� ������������ �� �� �� ������ ���� ���� ��� ����� ��������������
�� �� ��� ��� ��� ���������� ���� ���� ���
���������� ���� ���� �� ������ ���� ������� ���� �� ��� ��� ������� �� ��� ��������� �� ��� ������ �� ������� �
�� ��� ����� ��������� ���� ������� �� ����� ���� ����� ��� �� ��
Passage B ��� ������������ ���� �� ������� ��� �� ��� ��
���� ������������ ���������� �������� ��� �������� �������
����������� ������������ �� ��� ��� �������� ����� ��� ��� �������� �� ��� ������ �� ���� �����������
������ ��������� ������ ���� �� ��� ������������� ���� ��� ������� ���� ��� ������ �� ���� ����������� ��
���� ��� ����� ������ �� �������� ������������ �� ������ ��������� �� ���������� ����������
���� ��� ����������� �� ��� ����� �� ��� ���� ���� ��� ������� ���� ��� ������ �� ���� �����������
���� ������� �������� ���� ��� ������� �� ���� ���� ��� ��� ���� �� ��������� �� ���� �� �����������
������� ���� ������������ ������������ �� ��� ����� ����
�� ���� ������� ����������� ������ �� ���� ��� ��� ��� ������������ �������������� �� ��� ���� ���� ���
����� � ������ �� “���� ������������” �� ������� �� ��� ����� ������ ��� �� ��������� ���������������
����������� ����� ���� ��� ��� ��������� ��������
���� ���� ���� �� � ������������� ������ � ������ ��� �����
��� ������� ���� ������������� ������� ���� ������� ���
����������� �������� ���� ������� ��� ����� ���� ���
������ �� �������� �������� �� ���’� ��� ���� �������
��� �� ��������� ���� � ���������� ����������� ������� GO ON TO THE NEXT PAGE.
���� ��� ����� ���� ��� ������ �� �������� �������� ���
�������� �� ������� �� ��������� ������� ���
���������� �� �������� �� ������� �� ������� �� ��
�������� �� �� ������ ��������� ���� �� ������������
���� ������� �������� � ���� ������ �� �� �� �� ��
���� ��� ����� ���� ���� � �������� ����� �� ��������
������ ��������� ����� ��� ���� ������� �������� �
����������� ������ �� �� �� �� �� ��� ����� ���� ����
���� ��
���� ������ ���� ������� ��� ���� �� ���� �� ����
���� ��� ��� ������������ �� �� ��� ��� ��������� �� � �����
��� ��� ������ �� ��� ����� ���� ���������� ������� ��
������ �� ����� �������� ���� ��� ��� ���������� �������
��� �����’� ����� ��� ����� ���� ��� ������� ���� �����
�������—��� ������ ����� �����—���� ��������� ����
���� � ������������ ����� ��� ��� ������������ ��� ���
��������� ����� ���� ������ ���� ��� �� �����������
74

3 3 3 3 -25-
3
��� ������� � ������� ���� ������� � �� ���� ������� � ��� ����� ��� �� ��� ��������� ��������� �� ���� ���������
�������� �� �������� ������ ��� ����� �� ��������� ���� �� �� ��� �������� �������� �� ������� ��
����
��� ���� ���� ���������� ��� ������ ����
��� ������� ������������ �������� ���� ���� ������� ����
��� ���������� ����� ��� ��� �� ��� ��� �� ��������� ����������
��� �������� ���� ���������� ���� ����� �������� ����
��� ������ ������� �������������
��� ��������� ��� �������� ������ ��������� ������� ����
������ �� �� ������� ��� �������� ��������� ���
������������� ������� ���� ���������� ����
���� �� ����� ������ ���� ���������� ��������
����� �������� ������� �� ������� ������ ��� ��
���� �� ������� ����� ���������
��� ��� �������� ������� ��� �������� ����������� ��
���������� ������� �� ����� �� �������� �������
����� ���� ����� �� ������������ ��������
�������� ��� ������ ��� �� ������ ��������� ��
���������
��� ����� ������������ �� ��������� ���� �� ����� �����
�� ����� ����� ��� ��������� ���� ���’� �������
����� �� ��������� ������������ ��� ������� ��
������� ����� ������������ �� ��������� �����
���� ����� �� ��������� ������� �� ���������
�������������
��� ����� ���������� ���� ��� ����� ������ ����
������� ���� ���’� ��������� ����� ���� ��������
������� ��� ������ �� �������� ������� ����
���� ��������� ��� ��� ������������ ����������
�� ��� ������� ������

GO ON TO THE NEXT PAGE.


74

3 -26-
3 3 3
3 3
4HIS PASSAGE IS ADAPTED FROM A REVIEW OF A  BOOK ��� ��������� �� ��� �������� ��� ������ �� ������ �����
�������� �������� �������� ���� ��� �������� ��
�� � ������ ������ ����� ����í� ������ ���� �� ���
���������������� ������ �� ���� ��� ������ �� ������
������ ������ ������� ���� ��� ���� ��� ����� ��
����� �������� ��������
��������� ��������� �� ����� ��� “������� ��������
����������” ��� ���� ������� ��� ����������� ������� ��� ������ ��� ��������� ����� ���� ������� ���� �����
��� ���� ������� ���������� ���� ����������� ������� ������ ������ ��������
�������� �� ��� ���� �� ���� �� ��� ���’� ���� ��� ������ ��� ����� ������ ��� �������� ������ �� ���
��������� ��������� ����í� ���� ������� ���������� ������ ������
�������� ���� �� ��� ����� ��� ����� ����� ��������� ��� ������� � ���� ������� ������ ������ �����������
����������� ���� �� ��� ������� �������� �� ��� ���� ������
���� �������� �������� �� ��� ����� ��� ������ ��� ������ ��� ���������� ������� ��������� �� ����� �������
�������� ������� ���� ��� ������ ������ ���� �������
������ ������ �������� �� ��� �������� �� ��� ���������� ������� ��������� �� ����� ���
�������� �� ����������� ��� ������� �������� ������� �� ��� ������ ������
����������’� ��������� ��������� �� ��� ��������
���� ����������� ������� �� ���������� ��� ������� ������ ��� �� ��� �� �������� ���� ��� ������� ���� ����í� �����
�� ����������� �� ���������� ��������� ����� ���� �������� ����� ���� ����� ��� �� ��� ���������
������� ������ ���� ����� �� ������� ��� ���� ���������� ����� ��� ������� �������� ��������� ���������
������������ �� ����� ������������ ������ ���� �� ��� �� ��� ����� ��� ������
������ �� ������ ����� �������� ��������� ��
��� ���� �� ����� �������� ���� ������� �� ����� ��
���� ������������ ���� ���������� ������� ��������� ��
��� ������� �������� ��������� �� ��� �����
������ � ����� ������ �������� �� ������������ ���� ���
��� ������
������ ������ ��������� ��� �������� ����������� ����
��� ���� ���� ���� ����������� ������� ���� ���
����� ������������� ������� �� ������������� ���� ��
������� �������� ��������� �� ��� ����� ���
��� �������� �� ���������������� ������� � ���������
������
���� ����� ���� ��������� ��������� ��������� ��� �����
��� ���� ���� �� �������� �� ��� ������� ��������
������ ��� �������� ������ �� ��� ������ ������� ����í�
��������� �� ��� ����� ��� ������
������������ ����� ����������� ��� ��������� ���� ��
��� ���� �� ���� ��������� ��������� ��������� ���
�������������� ����� ���� ��� ����� �� ������ ��
����� ������ ��� �������� ������ �� ��� ������
��������� �� ����� �������� �� ������� ������� ���
�������
���� ����������� ��� ��� ���� ���� ���� ��� ������
��� ���� �� ���� ���� ���� ���������� �� ����������
������������� ���� �������� ������ ��� ������ ����
���� �� ������� �������
������ ������ ����������� �������� ��� ��� �������� ��
������������ ������ �������� ����������� �� ���������
���� ������� �������� ��������� ������� ����� ����
���� ��� ���� ������������� ��� �� ��������� ��� �� �������
��� ������ �������
������� ����í� ��� �� ������������ ��� ������ ��
GO ON TO THE NEXT PAGE.
����� ��� ����� �� ����� ��������� ���� ��������������
�� ��� ������ ������� ���������� �������� �� ���
���� ������ ������ ������ ���� ������� ������ ���� �� ����
��� ���������� �� ��� ������� �������� ����������
���� ��� ���� ���� �� ��� ������ ������ ���
������������� ���������� ����í� ������ ���� �������
���� ��� ���� � ��� ���������� �� ������� ��������
���� ������� ��������� ��� ���� ��� ���� ������������ ���
����� ���� ����������� ������ ���� ��� ������������
���������� �� ����� ���������� �� �������������� ���
�� ��� ��������� �������� �� ����� ��� �� ��������
����� �������� ��������� �� ����� ���� ���������� ��
���� ������� ���� ����� ������ ��� ��� ������ ������ ���������
�� ������� �������� �������� �� �� ��� ����� ��� ���
���� ������� �������� ������ ����� ���������� �������
� ������ ������������� �� ��� ��������� ������������ ��
��������� ��������� ���� �� �������� �� ������������
���� ��� ����� �� ������� ����������� ��� ���������������
��� ��� ���������� �� ������ ������������� �����
��������� ���� �� ������� ��� ������ ������ ���� ��
�������� �� ��� ���������� �� ������� ��������� ����
�� ��� ������ ������ ����������� �������� �� �� ��������
���� �� ��� ������ ������� ����������’� ��������� ���������
74

3 3 3 3 -27-
3
��� ��� ������� �������� ���� ����í� ������� ����� ��� �� ��� ��� ������ �� ��� ������� ��������� ����������� ����
��� ��������� �� ���� ���� ���� �� ������� ��������� ������ �� ����� ��� �� ��� ����������
������� ���� ��� �����
��� ������� �� ��� ��� ��� ������ ���� ��� ��������
��� ��������� ������ ������������� ����� ������� �� ��� ������ �� ������� ��������� ���� ��
��������� ��������� ��� �� �������� �� ��������� ��� ������ ������ ��� �� �� �������� �� �������
�������� ����� ����� �������� ��������� ��������
��� ��������� ����������� ����� ������� ��������� ��� ������� �� ��� ���������� ��������� ����í� ����
���� ������ ������ ������� ��������� ��� �� ������� �� ����� ����������� ����� �������
�������� �� ��������� �������� ����� ����� ��������� ��� ���� ����������� ������ �������
��� ��� ������������ �� ���� ������� ��������� ���� ��� ����
���� ������ ������ ������� ��������� ��� ��� ������� �� ��� ����� ��� ������� ���������
������� ���������’ ���� �� �������� �� ��������� ����� ������� �������� ��������� ���������
��������� ������� ���� ��� ����
��� ���� ������� ��������� ���� ����� �� ��� ��� ������ �� ����� ��� ����� �� �������
��������� ��������� �� � ����� �� ��������� ���� �������� ��������� ���� ������ �� ��� ������
����� ������ ��� ��� ������ ������ ��������� �� ������� ���������� �� ��� ������ ������
������� �������� ��� ��� ������ �� ��� ������������ ������� ��� ������
��� ���� ������� ��������� ���� ����� �� ������ ����� �������� �������� ��� ���
�� ��������� ������� �� ����� ���������� �� �������� �� ���������������� ������
�������������� ��� ��� ����������� �������� ��
����� ��� �� �������� ��� ��� ������� �������� ����� ��� �� ��� ���������
���������� ����� ������ ������������� ����� �������
��� �� ��� �� �������� ���� ��� ������ �� ��� ������� �������� ����������
����� ��� �� ��� ��������� ����� ��� ������� ��������
��� ������ ������������� ��������� ���� �����
��������� ��������� �� ��� ����� ��� ������
�� ������� ����������� ��� ��������������
��� ����� ������ ���� �� ������� ������ ���� ���������
���� ���� �������� ���� ��� ������� �������� ��� ������ ������������� ��������� ���� ��� ������
��������� �� ��� ����� ��� ������ �� ������� ��������� ���� �� ��� ������ ������
��� ����� ������ ���� �� ������� ������ ��� ��� ����������
�������� ����� ��������� ������������ ��� ������ ������������� ��������� ���� ��� ������
��� ����� ������ ���� �� ������� ������ ������ �� ������� ��������� ������������ ���� ���
���� ����� � ��������� �� ����� �� ����� ��������� ������� �� ��� ������ ������ ����������
������������ ��� ���������� �� ��� ��������� �� ������� ��������
��� ���� ���� ���� �� ���� ������ ������� ������� ������� ���� ��� ������� �������� ����������
����� �������� ������������ ��� ����� �������� �� ����� ������� �� ���� ��� ���������� �� ������
�������� ������������ ��������������
��� ���� ��� ��� ������� �� ����� ��������� ����� ��� ���������� �� ����� �� ������� ����������� ���
��������� ����� ������� �� ����� ��������� �������������� ������� �� ���� ��� ���������� ��
��������� ������ ������ ��������������

S T O P
IF YOU FINISH BEFORE TIME IS CALLED, YOU MAY CHECK YOUR WORK ON THIS SECTION ONLY.
DO NOT WORK ON ANY OTHER SECTION IN THE TEST.
74

4
-28-
 
4 
4 
44 4 4 4 4 4
4
SECTION IV
Time—35 minutes
26 Questions

Directions: The questions in this section are based on the reasoning contained in brief statements or passages. For some
questions, more than one of the choices could conceivably answer the question. However, you are to choose the best answer; that
is, the response that most accurately and completely answers the question. You should not make assumptions that are by
commonsense standards implausible, superfluous, or incompatible with the passage. After you have chosen the best answer,
blacken the corresponding space on your answer sheet.

�� ����� ����� ����� ��� �� ������������ ���������� ��’� �� ��� ������ ���� ��� ��� ��� ���� ����� ��� ��� ��������
��������� ��� ��� �������� ���� �� ���� ������ �� ����� �� ��� ������� ������ �� ���� ����� ������ ���
������������� ���� ���������� ���������� �� ���� ����� ������ ����� ���� ���� �� �� ���� ������ ���� �� ���
����� ��������� ����������� ��� ����� ���� ��� ��������� ����� �������
������ ��� �� ��� ��� ����� ���� ������� ����� ���� ���� ��������� �� ����� ������ �������� ����� �����
�������� �� ��� ���� ��� ������� ������� ������ ������ ����� ��� ������� ����� ��� ������� �����
��� �������� ���������� ���� ��� ������ �����
�����’� �������� ��������� ���� ��� ����������� ����’�
������� �� ���� ���� ����� ��� �� ��� ���������� �� ����� ���� ����������� ���
���������
��� ��� ���� ����� ��� � ������ ��������� ��� ����
��������� �� �� ��� ���� ���� ������� ���� ���� �� �� ���� ����� ���
��� ���� ���� �����’� ������� ����� ��� �� ������� ���� ���������� ���� ��� ������ �����
��� �� � ���� �� ��� �������� ���� �� �� ��������� ��� ����� ������ ������� ����� ���������� ������
��� � ������ ���������� ��� ������ �� ����� ��� �� ��� ���� �� ��� ������ ������� ���� ����� ������
��������� ������� ����� ��� ������� ��� ������
��� ��������� ������ �������� ������� ���� �� ���� ��� ���� ������� ���� ��� ��� ������ �� ���� ���� ���
�� ��� ������� ����� ��� ���� ��� ���������� ���� ��� ������ ���� ��� ����
��� ���� �� �����
�� ���������� ������ ��� ���������� ����������� ��� ��� ����� ��� ����� ������ ������� ���� ��� ���� ��� ����
���� �� �������� ��� ������������ �� ��� ������� ������� ���� ���� �� �� ���� ������
���� �� � ������ ���� ���� ��� ���������� ���
���� ������������ �������� ������ ���������� ��
��������� ��������� ���� ���� ������ ��� ����
�������� �� ��� ������ �� ������ ���� �� ���� ����
��� ������� ��� ���������� ������������ ���� ��
�� ����� �� ������ �� �� ��� ������ ������ ���� GO ON TO THE NEXT PAGE.
����������� ����� ���������� �������� ������������
���� � ������ �� ������ �������� ����� �� ���
���������� �����������
��� ������� ���������� �� ��� ���������’� �������� �� ����
��� ������ ��� ���������� ���������� ��� ������������
�� ��� ������� ���� �� � ������ ���� ���� ���
����������
��� ��� ���� ���� ��� ������������ �� ��� �������
���� �� � ������ ���� ���� ��� ���������� ������
��� ���������� ���������� ��� �� � ��������
����������� �� ������ ���������
��� �� ��� ������ ������ ��� ���������’� �����������
����� ���������� �������� ������������ ���� �
������ �� ������ �������� ����� �� ��� ����������
����������
��� ��� ����������� �� ������ ��������� ���� ��������
������ ��� ���������� ���������� ��� ��� ��� ��
��� ������ �� � ������ �� ������
��� ������ ���� �� ���� ���� ��� ������ ��� ���
������ ���������� ����� �� ��� ����������
����������
74

4
4     4
44 4 4 4
4 4 -29-
4
�� ��������������� ��� ���� �������� ���� ��� �������� �� ���������� ����� �� � ��� �� ������� ���� ����� ��������
$IPLODOCUS ������� ��� ������������ ���������� ���� ��� �� ����������� ��� ����������� ������� �������
�� ������� ������ �� ������� ��� ���� ���� ����� ��� ��� ���� ������ �� ���� �� ���� �� �� �����������
��� �������� ������ ���� ����� ���� ��� ��������� ���� � �������� �� ����� �������� �� �����������
�� $IPLODOCUS’� ���� ����� ����� ���� ��������� ��� ������� ������� ���� �� ��� ������� ������
���� ��������� ��� ���� ������ �������� ���� ������� ������ �� ���� �� ���� ������� ����
�������� ��� ���� ������ ����� ������ ������ �������� �������� ������� �������
$IPLODOCUS �� ������ ���������� ���������� ���� ���
��� ���������’� �������� �������� �������� ����� ��� ��
����� ����� $IPLODOCUS ���� ���� ��� �� ������ �� ��
��� ����������
���� ��� ������� �� �����������
��� ��� ���� ������ �� ������� ������ �������� ����
����� ��� �� ��� ��������� �� �� ���������� �������� ��
������ �� �� �� ������ �� ����� �������� ���� ��
��� ���������
�� ��� � ������ ������� ������
��� ��� ���� ���� �� ���� ��������� �� ����� �� ��� ����� ������ ��� ������� ��� ������� �� �����
������ �������������� �������� �������� ���� ���� �� ������ �� ������� �������
��� $IPLODOCUS ��� ��� ���� �� ��� �� ����� �� ������ ��� �� ����� ���� �������� ����������� �������
������ ��� ���� ��� ������ ������� ��������� ������� ���� ���� ������ �� ����� ���������
��� �� ����� �� ���������� ��� � ����� ������ ���� �� ��� ����������� ������� ������ ����� � �������
$IPLODOCUS �� ������ ����� �� �� �������� ������ ������ �� ����� ������ ���� �� ����� ��������
��� $IPLODOCUS ��� �� ����� ��� �� ��������� ��� �� �� ������ �� �������� ��� ����� ��������� ����
������������ ����������� ���� �� �� ������ �� �� ����� ������� ������ ���� �� �� �� ������ �����
��� ���� ����� ����������
��� $IPLODOCUS ��� ��� ���� �� ������ ���
���������� ���������� �� �������� ������ ������ �� �� � ����� �� ����� �������� �������� ��������� ��� �������
�� ����� �� �� ������� ���� ����� ����� ���� �� ��� ������ ����� ����� ������� �� ���� ���
������� ���� ����� ���� ������� ���� ���� ������ ��
�� ���������� ��������� �������� ��� ������������� �� ���������� ���� ���� ��� ��������� ���� ���� �����
����� ��������� �� ������� ������ ������ �������� ��� ���� ���� ���� �� ��������� ���� ����� ��������
���������� �� � ��������� ��������� ���� ���� �������� ���� ���� ����������� �� ���� ��������� ����
��� �������� ��������� �� ����� ���������� ��� ������� ���� ���’� ������ ������ ���� �� �� ���������� ��
���������� ������ ��� ������ ���� �� ����������� �������� ���� ��������
��� ������ �� ������ ����� �� � ������ ���������� ��
����� ��� �� ��� ��������� ���� ��������� ��������� ���
������ ���������� �� ���� �������� ���� ����� �����
���������
������� ������ �� ������
��� ������ ��������� ������� ��� ������ �� ���� �
����� ��� �� ��� ��������� ����������� �� ������ ����
����� ������� �����������
����� �� ������� ��� ��������� �� ��� ���������� ��������’�
��� ��� ���� ����������� �� ����� �������� �� �� ����
���������
������ ����������
��� ��������� ������ ��� �� ������� �� ������� ��� ��� �������� �� � ����� �������’� ��������� ��
������ ������ ��� ���������� ������� ���� �� ������� �������� �� ����������� �����������
����������� ���� ��� ������� ����� ��� ��������� ���� ����
��� ��� ������������� �� ��������� �� ������� ������ ������� ���� �������
������ ���������� �� ���������� ������ �� ���� ��� ������ �� �������� �� ������ ������ ���� � ����� ��
���������� ���������� ������� ��� ��� �������� ������� ����� ���� �� ��� �����
��� ���������� �������� ������ ��� ������ ������ ��� ����� �������� ��������� ���� ������� ����� ����
���� �������� ������ ����� ������ ��� �������� �� ��� ����� ��� ��� ������������ ���� ���� ���
��������� �� ����� ���������� ��������� ��� ���� ������ �� ������� ������� ����
��� ��� ���������� ������ ��� ������ �� �������� ���� ��� ���� �������� ��� ��� ��� ������������ ����
��� ���� ������ �� ������ �� ������������� ����
����� ���� �� ������� �������
��� ��������� ������ �� ����������� ���� ����������
�� ��� ���� ���� ��� ��� �� ��������� ������� ��
�����������
GO ON TO THE NEXT PAGE.
74

4
-30-
 
4 
4 
44 4 4 4 4 4
4
�� ����� ��� ����� �� ��� ��� ��� ���� ����� �� ��� ������ ��� �� ����� �������������� ���� ���������� ���� ���������
����� �� ��� ���� ���������� ���� �������� ��� ������� ���� ��� �������� ������� ��������� ���� ������ �� ���
������� �� ���������� ������� �������� ��� ����� ������ ����� �� � 4YRANNOSAURUS REX ��������� �������� �� ���
����� ������� �� ���� ���� �� ����������� ��� ��� � �������� �������� ���� ��� 4 REX ������ ���� �� ��
������� �������� �� ��� ������� ������� �� ��� �������� �������� �� ���������� ���������
����������’� ��������� ��������� ���� �� ��� �������� ��
�� ����� ��� �� ��� ��������� ���������� �� ��� ��������
�������� ���� ��������� ��� ������� ������� �� ������
���� �����������
��� ������ �� ����� ��� �� ��� ��������� ���������
��� ��� �������� ����������� ����������� ����
����� �� ���� ������ �� ���� �� ����� �� �������� ���
��� �������� �� � ��� ����� �� ������� �� ���
���������
�����������
��� ��� �������� ����� �� �������� ��� ����������� ��� ��� ���� �� �� �� ���� ��������� ���� ������ �� ���
���� ������� �� ���� ���� �� ��� ������ �� ���� ����� �� � ���������
�������� ��� �� ������ ��������� ��� �� ����� ��� �������� �� ��� ������� ��� ����� ����
��� ��� ���� “��������” ��������� ������� ������� ��������� ��� ������� ������� �� ������
������ ��� ������ �� ��� ��������� ��� ��� ������ �� �� ��� ������� ���� ��� ��� �������
��� ��� �������� ��������� ������� ������ ������� �� ���� ����� �� ���� ������� ��������
�������������� ���� ��� �������� ��� �� ��������� ���������
�� ��� �� ���� �� ���� ��� ����� ��� �� �� �������� ���� 4 REX �� ���� ������� ������� ��
��� ��� �������� ����������� ������� �� ��� ��������� ���������� �������� ���� �� ������� ����� �����
�� ������� �� ����� �� ������� ��� ����������� �� ����������
��� ������ ����������’� ���������� ��� �����������
�� ������ ��� ������� ���������� ��� ��� ����� ���� ����� ������� ���� ��� �������� �������� �� 4 REX ���
��� ��������� ����� ������������ ������� ���� ������ �������� ����� �� ��������
���� ��� ��������� �� ����������� ������������ ���� ����
������ ����� ������� �� �������� �� ��� �������� �������� ��� � ���������� ��������� ����������� ����� ���������
��������� ��� ��������� ���������� �� ���� ���� ��� ����� ������������ ������ ����� ����� �������� ����� ������
���������������� ������ ������ ���� �� ���� ������ �������� ������� �� � ����������� ��� ����� ��������� ����� ����
������� ����� �������� ������ ���������� ��� �������� ���� ��� ���� ���� ��� ���
���������� ��� ��� ��� ���������� ������������� ���
����� ��� �� ��� ���������� �� ����� ����� ���� ���� ��
������� ��� ����������� ������ ����� ������� ����� ��
������� ��� ������ ���� �� ���� �������� ������� �����
��������� ��� ����� ��� �������� �� ����� ����� ��������
������� ������� ��� ����� ��������� ������
��� ���� ����� ����� ������ ������� ������ ��������������
��� ����������� ����������� ����� �� ������� ��� ���� ��� �������� ���� ���� �� ��������� �������� �����
��������������� �� ������������
����� ��� �� ��� ��������� ���������� �� ���� ��������
��� ���� ������ ��������� ���� ��������� ���� �� ��
��������� �� ��� ����������� ������
����� ���������������� ������
��� ��� ���� ����������� ���������� �� �������� ��� ��� ��� ���������� ������� �� ���������� �����
��� ���� ������ �� �� �� ������ � ������’� �������� ����������� ��� ���� ���������� ���� ��� ���
��� ����������� ����������� �� ���� ���� ������� �� ����� ���������
�������������� ���� ����������� �� ���������� ��� �� ��� ��� ������ ��� ������� ������� �� �����
��� ������ ���� �� ���� ����� ������� ����� ����� ����������� �� � ���������� ������ �� ���� �� ����
������ �� ����� ��������� �������� �� ����� �������������� ������ ����
������� ��� ����� ����������� ��� ���� ������ �� ����������’
��� ����������� ���� �� ���� �� ��� ���
����������� �� �������
��� ���������� ����� ����������� �� ��� �� ������������
�� �������� ����� ������������
��� ���������� �������� �� � ������� �������� ���� ��
�� ������ ��������� �� ����� ���������

GO ON TO THE NEXT PAGE.


74

4
4     4
44 4 4 4
4 4 -31-
4
��� ����� ��������� ��� ������’� ���������� ������ ���� ��� ����������� �� �� ��������� ��� ����������� �� ���—�� ���
�������� �� ���������� ��� ����� �� ��� ������’� ��������� ������ ���� ��� ������� �� ���������
������ ���� ���������� ��� ����� �� ������ �� ��� ��������—�� ��� � ������ �������� �����������
��� ����� ������� ���� �� ������� ��� ���� ���� �������� �������� �������� ����������� ��
��� ������ �� ����� ������� ��� ����� �� ����� �� ���������� �� ����� �� ������ ��� ������������
��� ������ �� ��� ������� ����� ����� ���� �� �� ���� ����� ���� ����� ���� ������ �� �����������
����������� �� ���� ��������� �� ������� ��� ����� ��������� ��� ��������� ���������� ��������
�� ��� ������’� ������ ���� �� ������� ����� �� ���� �������� ����� �� ��������� ��� �����������
����� �������’ ������� ������� ������� �� ������� � ����� ������ ���
�������� ����������� ���� ����� �� ���������
����� ��� �� ��� ��������� ����������� �� ������ ����
������� �� �������� ���������� � ����� �������
����� �� ��������� ��� �������� �������� ����� ��� �����
��������’� ������� ��� ���������� ������ ��
��� � ������’� ���������� ������ ��� ������� �� ��� �������� ��� � ���������� ������� ��� ���
������� ��� ����� �� � ����� �� ������ ������ ����� ����� �������� �� ����� �� ���� ���� �
��� ������������ �� ����� ������’� ����� �� ���������� ����� �� ����� ������������ ������ ���
����������� ���� ��������� ���� ���� �� ��� �� �����
����� �����’� ������ ��� ��������� ���� ��� ���������� ��������� �
��� � ������’� ���������� ������ ���� �������� �� ������������� ����������� �� �������� �� �������
���������� ��� ����� �� ��� ��� ������ ���� �� � ����� �������� �� ��
���������� ��� ����� �� ������� ������’� ������ ��� �������� � ������� ��� ���� ������� ���� �����
���� �� ��� ��� ������ ��� ���� ������ ���� ��� ���� ���������� ��� ������� ������� �� ��� �� ���
����� ������’� ������� ����� ����� ����������
��� ��� �������� � ������’� ���������� ������ ����� ��� ��������� �� � �������������� �� ��������� ��
�� ���������� ��� ����� �� � ����� �� ������ ������� ��������� ���� �������� �� �������� ���
������� ������ �� ��� ����������� ��������� �� ���������� �� ������ �� ������
�� ��� ��� ����� �� ����� ������ �� �� ���������� ��� ���������� ��� ��������� � ����� ��������� ��
��� ���� ��� ������ �� ��� ������� ��� ������� ��������� � ���� �� ����� �� ������� �� ��� ��
������� ��� ����� �� ��� ������ �� ���� �� ����� ����� �� ���� ��� �����
������� ������ �� ��������� ��������� �� ���
������’� ��� ������������ ��� ���������� ���� �� �� ���������� ��� ������� ����
��� � ������’� ���������� ������ ���� �������� �� ��� �������� ����� ������ �� ������� ������� �� �����
������������ �� ��� ����� �� � ����� �� ������ ��� �� ��������� �������� ������� �������� �� ����
��� ���� ��� ����� �� �� ����� ��� ��� ���� �����’� ����������� �� ������� ��� ������� �� �������
����� �� �� ���������� ������� �� � ����������� ����� ��� ��������� ����� ��
�� ������ �� ����� �������� ����� ��������
��� ������ �� ���� ��� ������������� ������ ����� ������
��� ��������� �� ��� ���������’� �������� ��
�� ���� ���� �� ����� �� ����� ���� � ������ �� �
������������ �� ���� ��� ��������
������������ ����� ���� �������� ����������
��� �� ��������� ���� ���� ����� ������������ ��� ������ ������ �� ��� ��������� �� �������
��� �������� ���� ������������ ���� �� ��������� ��� �������� �������� ������ ���� ���� ����������
��� ������� ���������� ��� �������� ��� ������� ��
��������� ���� ��������� ������������� ���� �� ��� �������� ���� � ���� �� ��������� ��������
��������� ������������ ������ �� ��� ���� �������� ������ �� �����
�������� ����� ���
�� ��� �� ��� �����’� ���������� ��� ����� ���� ����� ���
��� ����� ���� �� ������ ��� ��� �� �����������
�� ��� ��������� ���� ���� �� �����
�������� ������ �� �� ��� ���� ��� ��
��� ��� ������������� �� ����� ������������ �� ����� ����������� �������� ������
��� ��� ��������� �������������� ��� ��������� ���� � ������� ��������� �� �������� �����
��� ��� ��������� ������������� ���� ��� ����� ������� ����� ���� �� ������������� �������� ������
����� ���� ���� ���� � ������ ������ ���� ��� ���� ���� ��� ������� �� �������
��� �� ������ ���� �� ����� ���� ���� ���� � ����� ������� �� � ����������� ����
������ �� �� ������� �� � ��������� �������������
��� � ��������� ������������ ���� �������� �� �������
�������� ���� ���� ��� ������� ����� ���� ����
���� � ������
��� �� ������ �� �� ����������� ������������ ����
�� ����� ���� ���� ���� � ����� ���� �� ��� GO ON TO THE NEXT PAGE.
������������ �������� ���
74

4
-32-
 
4 
4 
44 4 4 4 4 4
4
��� ���� ������������ �� ������� �������� �������� ��� ����������� � ����� ����� �� ������ ���� ��–�� ���
���������� �� ����� ������������ �� ���� ������ ��������� ��������� ���� �������� ������ ���� ���� ��
���� ��� ������� ��� ����� ��� �� ��� ��������� �� ����� ��� ��������’� �������� ��� ������������� ����������
������ ����� ������� ���� ������� ������ �� ����������’� �� ��������� ���������� � ������� ��������
���� ��� ������������� ���������� ������������������� �� ��� ������ ������ ����� ����� � ���� �� ���
������� ��������� ��� ���� ��������� ������ ������� ���������� �� ��� ����’� ���������� ������ �����
��� ������� �� ���� �������� �� ������������ �� ������ ��� ������ ������ ���� �� ������������� ��
������’� ���������� �� ������� ��������� ����� ���� ���� ��������� ����������� ���� ��� ������ �����
��� ��� �������� �� �� ���������� �������� �� ���� ������� �������� ���� ��������� �� �� ���� ��� ���������
��������� ������ ������ ��� ������������� ��������
��� ���������� �� ��� �������� ��� �� �������� ����� �� ��� ����������’� �������� �� ������ �� ���� ��
����� ��� �� ��� ��������� �� ��������
��� ������ ���� ��� ������ �� �� ������ ���� ��� ������
��� ��������� ���� ������ �� � ������ ���������� ������ �� �������� �� ������� ���� ������
��������� ����� ���� �� ��� ���� ��������� ��� ������ �� ��� �������� �� ����������� ��� ���
��������� ������ �� �� ������
��� � ������� ��� ����� ��� �� ��� ��������� �� ��� ������� �� ����� ��� ��������� �������� ��� ���
����� ��� ������ ���� ��� ��� ���� ������ �� ���� ���� �� ���� ��� ����������
���������� � ������������ �������� ��� �������� �� ������ �� � ���������� ���� ���
��� ������� ��� ��� ������ ���� �� �������� ����� �� �����
��� ����������� �������� ��� ������ �� � ������ ���� �� ����������������
��� � ��������� �������� ������� ���� ��� ������ ���
����� �� �������� ������� �� ������������
��� ��������� ������ ������ ��� ������������ ���
��� ������� �� ����������’� ����������

��� ����� ��� ����� �������� �� ���� �������������� � ���� ��� GO ON TO THE NEXT PAGE.
������������ ������ � ��������� �� �������� �������� � ���
�� ��� ���� �������� ���� ��� �� �� ������� ���������� �����
���� ���� �������� �� ���������� ������� �� ����� ������
������� ���������� ��� ��� �������� ������ ����������� ��
������’ ������������ ���� ������ ��� ��������� �� ������
�� ������� �� ����� ����������� �� ����������� ���� ���
����� ���� ������� ���������
��� �������� ���� ����� ��� �� ��� ��������� ����������
�� ��� ������� �� ��������� ��� �������� ���� �� ����������
�� ��� ���������� �� ���� ��������������
��� ��������� ��� ������� �� ��� ���������� �� ���
��������
��� ����������� �� ������������� ������ ��� ���������
���� �� ������� ��� ��������
��� ���������� �� ���� ���� � ������� ��� ������� ��
������� �� ��� �������� �� �����
��� ���������� � ��������� �������������� �� �
������� ����� ���� �� ��� ��������’� ����������
��� ������� ���� ��� ����������� �� ��� �������� ���
����������� ������������
74

4
4     4
44 4 4 4
4 4 -33-
4
��� ��� ������ ������� ��� �������� ��� �������� ��������� ��� ������ �� ��� ������� ���� ���� ��������� �������� ���
������ ��� �� ����� ������������� ����� ��� ������� ����������� �������������� ����� �� ��� ������
��������� ��� ������� ���� �� ����� ���� ��� ������ �� ��� ���� �� ��� �� ����� ���������� ��� ���� �� ����
������ ��������� ��� ��� ������� ��� ��� ���� ���� ������� ��� ���� ��������� ������� ��� ���� ����
������ ��� ���� ��� ����� ������ �� �� � �������
��� ������� �� ��������� �� ����� ��� �� ��� ��������� ��
������ �� ��� ���������� �� ������ �������
���� ������� �� ���� �� ��� �������� ������
�� ���� �������� � ���� ������� ����������
��� ��� ������ � ������������ ������������� ���� ���� � ����� ������� ���� ������� ����� ��� ����
��� ������� ����� �������� ������� ��� ����� ���� ���������� �� ���� ��������� �������� ��
���������� �� �� ���� ��������� �� � ������ ����������� �����
������� �������� ����� ����� ��� ������� ��� ���
��� �������� �� ��� ������ ��������� ���� ����� ��� �� ���
������� ��� �������� �� �������� ��� ��� ��������
��������� �����������
����������
��� ��������� ������� ��� �� ����� ������������� ��� �� � ����� �� �������������� �� ������ ��� ��
����� ������� ���������� ���� ���� ��� ��������� ������������ ������ �� �� ������ ��
������ �������� ��� ��� ������� ��� ������� �������� �� �� ��������������� ���� ���������
���������� ��� ��� ������ �������� �� ��� ��� ��� ������ �� ��������� �� ������ ���� ��� �����
������� ���������� ��� �� �� ����� �� ����� ������� ���� ����������� �������� ���� �� �������� ����
������� ���������� ������� �� ������������ �������
��� �� ��������� ��� �� �� ����������� ���� ��� �� � ����� ������ ��� ������� � ������ �� ��� ����
���������� ���� ��� ������ �� ��� ���� ����� ��� ��� ��� ������ ��� � ���� ����������� ��� ������
�� ���������� �� �� ���� �������� ��������� ����� ������ �������� �� ����� ���� �������
��� ������ �� ��� ���� �������� ��� ��� �������� ��� ������ ��� ����� ���� ������� ������������
����� ����� ��� ��� ����� ������ ��� ������� ��������� ������ ��� ��������� ���� ���� ������
��� ��� ������� ���� ���� ��� ������’� ���� ����� ��� ���� ��� ������� ������������� ���������
������� ������� ��� ������ ������ �� �� ������� �� ��� � ��������� ������ ��� ������� � ������ ������ ��
� ������ ��� ���� �� ������� �������� ����� ����� �� ��������� �� �� ����������� �������
����� ���� ���� �� ����� ������� ��� �������
��� �������� ������ ��� ���� ���� ����� ����������� ��� �������� ����� ����� ��
��� ��������� ��������� ��� �� �� ������ �� ��� ���� ������ ���� ��� ���� ����� ���� ���������� �����
�������� ���� ���� ��� ����� ����� ���� ���� �� ��� ���� ������� ���� ����������� ��������������� ��� ����� ����
���� �� ���� ���� �������� ���������� ��� ����� ������� ���� ���� ���� ������ �� �������� �� �� �������
��� ����� ��� ���� ���� �� ��� ����� ���� �������� �������� �� ���� ��� ������� �������� ���� ��� ��������������
���� �������� ���������� �������������� ������� ���� ����� ��� ������� ����� ����
���� ��� ��� �������������� �������� �������� ��������
��� ���������� ������ ���������� ��� ����� �� ���������� �������� ��� ��� ������� �������� ����� ��� ��������������
��� ���� ����� �� ��������� ���� �� ���� ���� �������������� ������� ���� ���� �����
������������ ��� ����������� ����� ��� ����
��� ��������� �� ��� �������� �� ������ �� ���� ��
���������� �� ����� ��������� ������������ ���� ��
���������� ������� ������ ����������� ��� ������ � ����� ������ ���� � �����������
��� ������ ���� ���� ������� ��� �������� ��� �
��� ���������’� �������� �� ���� ���������� �� ���
���������� ����� ��� ��� ��� ���� ���������� ����
��������� ���� ��
����� �� �����
��� ����������� �� � ������� ��� ���������� ���� �� ��� ����� � ���������� ��������� �� � ��������� ���
�������� �� ��������� ��� ������ ������ ������� ��������� ��� ������ ��
��� ����� �� ��������� ���� ���������� ����� �� �� ���� ����� ���� �� ��� �����
�� ���������� ������� ������ ���������� ��� ������� �� ��� ��������� �� ���������� ��� ���
��� ������ ��� �������������� �� �� ������ ����� �� ��� �� �������������� �� ��� ���������� ����
�������� ��� ��� ����� �� �� ������������ �� � ��������� �� ��� �������� ����
�������������� ���� ������� ��� ����������
��� ����� �� ������� ��� ����������� ���� ��� ����������
��������� �� ���������� ��� ��� ���������� ��
������ ���������� ���� ������������� �������
��� ���������� ����� �� ��������� ���� ���������
���� ��� ����� ������ �� ��� ����������� ���� ��� GO ON TO THE NEXT PAGE.
���������� �� ��� �����������
74

4
-34-
 
4 
4 
44 4 4 4 4 4
4
��� ��� ������������ �� ������� � �������� ���� �������� ��� ��� ��� ������ �� ��������� � ������� ������� �����������
���������� ����� � ����������� ������� �� ���� ���� ���� ����� ���� �� �������� ������ ��� ������ ����������
������ ��� ��� ������� ����� �� ��� ������� �� ��� ���� �� ���� �� ����� ������ ���� ����� � �����’� ������ ��
������� ��� ������������ ���� �������� ���� �������� ��� �������� ���� �� ��� ���������� �������� �������� ����
���������� ����� � ����������� ������� �� ���� ������ ���� �� ���������� ����� ��������� �� ����� ��� �� �� ���
�������� �� ������� � ���� ���� �������� ��������� �� ��� �������’� �������� �� ���� ���� �� ������
��������� ������� ��� �� ����� ��� ���� ����� ��� �����
����� ��� �� ��� ���������� �� ����� ���� ������� ���
��� ���������� �� ������� � ���� ������� �� ����� ��� ���������
�� ��� �������������� ������������ ��������� ����� �� �� ��
��� ��� ��������� �� �������� ��� ��� �� �������� ����
����� ���� ����� ��� �� ��� ��������� �����������
� ������������ ������ ��� ������������� �����
��� ��� ���������� ���� ���� ��������� � ����������� ��� ����� ��� ���� ��������� ������� ����������� ��
�������� ��� ������� ��� �������� ��� ���� � ��� ������ ���� ��� ���� �� ��������� �� ���������
����������� ���� ��� ����� �������� �� �������� ��� � ����� �� ��� � ���������� ������ �� ���� ���
�� ������� � ���� ��� ��� ��� ��������� ������� ���� ������� ����������� �� �� ����� ����������
����� ��� ��� ��� ��������� �������� ����������� ��� ��� ������ �� �������� ������ � �������� ��� ����
��������� � �������� ����� �� ��� ������������ �� ������������ ���� ���� ���� �� ����� ���� ��
��� ��� ���������� ��� ������ �� ���� �������� ���� ��� ���� ����� ��������
������� � ����������� ����� �� � ������������ ��� ��� ���� ���� ���� � ������� ���������� ��������
�������� �������� �� ������� � ���� �������� ���� ������� ����������� ���� ��� ������ ���� ��
��������� �� ��� ������������ ���� �� ��� ��� �� ������� ���� ��������
�����������’� ���������� ��� ����� ���� ����� ��
����� ����������
��� ��� ���������� ���� ���� ��������� � �����������
��� �������� ����� ��� ������� ��� ���� ���
�� ��� ������� ������ ��� ��������� ��� �����
���’� ��������� ��������� �� �� ������� ���� ��� GO ON TO THE NEXT PAGE.
���������� ������ ��������� ������� ��������
�� ������� � ���� �������� ����������
��� ��� ���������� ���� ���� �������� ����������
� ����������� �������� ��� ������� ��� ��������
��� ���� � ������� �� ����� �������� ��
������� � ���� �������� ��������� ���
����������� ��� � �������� ������ ��������
��� �������������� ���� ���� � ���������� ��
��������� ��� �� ���� ��� �������� �� ������� ��
��� ������ �� ���� �� ��� ������� ����
��� ��� ���������� ���� ���� ���������� � �����������
�������� ��� ���������� ���� ����� �����������
������������ �� ��� ��� �������� ���������
�� ������ ���� �������� �� ������� � ���� ����
�������� ���������� �������� ��� ����
������������ ���� ������ ��� ����� ��� ����������
�������� ��� ����������
74

4
4     4
44 4 4 4
4 4 -35-
4
��� �� ��� �������� ��� ���������� �� ��� ����� ��������’� ��� ���� �������� ��� �������� �� ��� �������������
������� ��� ������������ ���� ���������� ������� �� ��� ���������� �� ���� ���� �� �������� � ������ �� ���
����� ��������’� ������� ��� ���������� ��� ������ ���� ���� �� ������� �������� ���� �������� ����� ���
�������� ������� �������� ���� ��� ������ ��� ������ �������� ������ �� ��� ������ ��������� ��� �����
�� ���������� �� ������� �������� �������� �� ��� ����� �������� ���������� ���� ��������� �� ��� ��������
��������’� ������� ��� ������������ ���� ����� ��� ����� ������ �������� ����� ������
����� ��� �� ��� ��������� ��������� �������� ������ ����� ��� �� ��� ���������� �� ����� ���� ����������� ���
��������� ���� �� ���� �������� �� ���� �� ��� �������� ���������
������
��� �������� ����� ���� �������� ���������� �
��� �� ��� ������ ��� ������� �������� ������� ��������� ���� �������� ������� ���� ���� ���
������� �� ��� ������� �������� ������� ���� �� ������� �������
������� �� ������� ���� ����� �� ���� ������ ��� ��� ��� ����� �� ��� ������ �������� ���� ��� ����
���� �� ���� �� ��� ������� ���� ������ ���� ������� ���� �� ����� �� ��� ���� ���������� �� �������
������� ���� ��� ������ ��� ���� �� ���� �� ��� �������� ������
������� �� �� �������� ������� �������� ������� ��� ���� ���� ���� ��� ������ �� ��� �������� ����
������� ������ ������ ����� �� ����� ��� ���������� ����
��� �� ��� ������ ��� ������ ��� ���������� �� ��� ����������
���� �� �������� ���� ��� ���� ��� ������ ���� ��� ���� ��������� �� ��� ���� ���� ����� ���� �������
����� ���� �� �������� ���������� �������� �� � ���� ������ �� ������ ����� ��� �����������
���� ������ �� ����� � ��� ������� ������ ���� ����� ��� ���������� �������� ������
��� ������ ��� ���������� �� �������� ���� ��� ��� �������� ����� ����� ���� �� ����� ������
���� ��� ������ �� ��� �������� ���� ��� �����
�������� � ��� �������
��� �� ��� ������ ��� ������� ���� ��� �������
������� ��� ��������� ���� ������� �� �������
���� ������� ���� ����� ��� ��������� ���������
��� ��������� ����� �������� ���� ��� �������
��� ��������� �� �� �������� �������� ���� ���
��� ��������� ��������� ��� ���������
��� �� ��� ������ ��� ������ ������� � ����� ��
��������� ��� ����������� �� �������� � ����� ����
���� �� ��� ������� ������� ���� ���� �������
���� ��� ������ ������� ������ �� ������ �����
������� ���������� ������� ��� � ������ �� ���
�������� �������� �������� ��� ������ ��������
��� �� ��� ������ ��� ������� ���� ����� ���� �� �
�������� ��������� ���� ������� �� �������
�� ��� ������� ������ �� ��� ������� � �����
������� �������� ���� ����� ���� �� � ��������
��������� �� �� �������� ��� ��� �������
������ �� ��� ������� � �����

S T O P
IF YOU FINISH BEFORE TIME IS CALLED, YOU MAY CHECK YOUR WORK ON THIS SECTION ONLY.
DO NOT WORK ON ANY OTHER SECTION IN THE TEST.
ACKNOWLEDGMENTS
Acknowledgment is made to the following sources from which material has been adapted for use in this test booklet:

Michael S. Gazzaniga and Megan S. Steven, “Neuroscience and the Law.” ©2007 by Scientific American, Inc. http://
www.sciammind.com/article.cfm?articleID=00053249-43D1-123A-822283414B7F4945.

Jeffrey Rosen, “The Brain on the Stand.” ©2007 by The New York Times Company. http://www.nytimes.
com/2007/03/11/magazine/11Neurolaw.t.html?_r=2&oref=slogin&oref=slogin.

Angus Trumble, “Smelly Masterpieces.” ©2008 by Times Newspapers Ltd.

Kipling D. Williams, Martin J. Bourgeois, and Robert T. Croyle, “The Effects of Stealing Thunder in Criminal and Civil
Trials.” ©1993 by Plenum Publishing Corporation.

36
COMPUTING YOUR SCORE
Directions:
Conversion Chart
1. Use the Answer Key on the next page to check your For Converting Raw Score to the 120-180 LSAT
answers. Scaled Score
LSAT Form 4LSN111
2. Use the Scoring Worksheet below to compute your Reported Raw Score
raw score. Score Lowest Highest
180 99 101
3. Use the Score Conversion Chart to convert your raw 179 98 98
score into the 120-180 scale. 178 97 97
177 96 96
176 95 95
175 0 0
174 94 94
Scoring Worksheet 173 93 93
172 92 92
1. Enter the number of questions you answered 171 90 91
170 89 89
correctly in each section. 169 88 88
168 86 87
Number
167 85 85
166 84 84
Correct
165 82 83
164 80 81
SECTION I.................. 163 79 79
162 77 78
SECTION II................. 161 75 76
SECTION III................ 160 74 74
SECTION IV................ 159 72 73
158 70 71
157 68 69
156 67 67
2. Enter the sum here: 155 65 66
154 63 64
This is your Raw Score.
153 61 62
152 59 60
151 58 58
150 56 57
149 54 55
148 53 53
147 51 52
146 49 50
145 48 48
144 46 47
143 44 45
142 43 43
141 41 42
140 40 40
139 38 39
138 36 37
137 35 35
136 33 34
135 32 32
134 31 31
133 29 30
132 28 28
131 27 27
130 25 26
129 24 24
128 23 23
127 22 22
126 21 21
125 20 20
124 19 19
123 18 18
122 17 17
121 16 16
120 0 15
*There is no raw score that will produce this scaled score for this form.

37
ANSWER KEY

SECTION I

1. C 8. A 15. E 22. E
2. C 9. E 16. D 23. D
3. B 10. E 17. A 24. B
4. C 11. D 18. A 25. D
5. C 12. E 19. A
6. A 13. A 20. E
7. E 14. D 21. B

SECTION II

1. C 8. E 15. B 22. B
2. D 9. A 16. A 23. C
3. E 10. A 17. E
4. A 11. A 18. D
5. E 12. C 19. B
6. E 13. E 20. B
7. B 14. D 21. A

SECTION III

1. D 8. D 15. A 22. E
2. E 9. C 16. D 23. A
3. A 10. B 17. B 24. B
4. A 11. D 18. A 25. B
5. B 12. A 19. B 26. D
6. B 13. D 20. C 27. E
7. B 14. E 21. C

SECTION IV

1. B 8. B 15. C 22. C
2. C 9. B 16. E 23. E
3. E 10. C 17. E 24. D
4. D 11. A 18. E 25. A
5. D 12. D 19. C 26. B
6. B 13. D 20. D
7. A 14. A 21. A

38
LSAT® Writing Sample Topic
©2014 by Law School Admission Council, Inc. All rights reserved.

Directions: The scenario presented below describes two choices, either one of which can be supported on the basis of the information given.
Your essay should consider both choices and argue for one over the other, based on the two specified criteria and the facts provided. There
is no “right” or “wrong” choice: a reasonable argument can be made for either.

� ������� ��� ������� ������ ��� ������� ��� �������� ������� �� ����� ��� �������� ������ ��������� ����� ��� ������� �� ����� ���
���� ��������� �� �� �������� �� ��� ������ ����� ����� �������� ������ ������ ����� ��� ����� ������ ����� �� ����� �� ����� ��� ����� ��� ���
������ ���� ��� ����� ����� �� ��� ��������� ��� ���������

ŇŇ ���� ���� ��� ����� ���� ���� �� ����� ����� �� ������� ��� ��������� ��������
ŇŇ ���� ���� �� �� ����������� ���� ������� �� ����� ����������� ����������� ��� ��� ������� ���� ���� ������� �����

������ ��� ������ ���� ���� ��������� ������� ��������� ���� ��� �������� ������ ���� �� ���� ����� ��� ������ �� ������ ����� �� �
���� ������ ���� ���� ���� ������ ������� ������ �� ��� ������ ���� ���������� ��� ������� ������� �� �� �������� ���� ���� ���� ���� �������
�������� �� ����� �� ���� ���� ��������� �������� ��� ������ �� ����� ����� ����� ���� �� ������� ����� ������� ������������� ����� ��� ���
���� ���� ����� ������� �� ����� ��������� ��� ����������� �������� �� ���� ���� �������� ����� ������ ����������� ��������

������ �� �������� ��� ��� �� ���������� ��� ��������� ��� ��� ���������� ��� ������� ������������ ���� ����� ���� �������� ���
������ �� ����� ����� ����� ��������� ��� ������������� ���� ����� ������� ���������� ��� ��� ��������� �� � ����� ����� ��� ������������� ����
���� ������������ ������ ��� ���� ������� �� � ������� ���������� ����� ��������� ��� ������������� ������ ���� ����� ���� �� ������� �� ���
������ �� �� ��������� �� ����� � ��������� ������’� ������� �� ������������ ����� �� ��� ����� ������ ��� ������ ���� ����� ���� �� �� ���� ����
�� � ����������� �� �������� ��� ������ �� ������ ��������

Scratch Paper
Do not write your essay in this space.

Вам также может понравиться